Está en la página 1de 57

1.

Femenina de 68 años de edad que viene a la sala de emergencias con una hernia
umbilical encarcelada evidente. Tiene distensión abdominal grave, clínicamente deshidratada y
reporta vómitos fecaloides persistentes durante los últimos 3 días. Aunque cansada, débil y
sedienta, ella está despierta y alerta y su sensorio no ha sido afectado. Los laboratorios revelan
concentración sérica de sodio de 118 mEq/L, ella no tiene condiciones médicas crónicas ni
toma medicamentos. ¿Cuál de los siguientes es la explicación fisiológica más probable para el
nivel de sodio sérico?
a) Ella tiene intoxicación aguda de agua
b) Ella ha estado vomitando y atrapando fluidos hipertónicos en el lumen intestinal
c) Ella ha vomitado y secuestrado fluidos que contiene sodio, y ha conservado
agua endógena e ingesta
d) Debe de ser un error de laboratorio, tal nivel de sodio sérico produciría coma
e) Depravación de volumen conduce a pérdida renal de sodio

Respuesta correcta: C
2. Femenina de 82 años que se presenta a sala de emergencias con dolor precordial. Ella se
despertó temprano esta mañana con un dolor precordial centralizado, sordo e inmediatamente
marcó el 911, ya que pensó que estaba teniendo un ataque cardiaco. Afirma que ha tenido
episodios similares antes, pero que ningún había sido tan severo. Comenzó con dolores
precordiales hace dos semanas, específicamente cuando camina alrededor del centro
comercial. Fue una fumadora de 45 paquetes por año, pero dejó de fumar hace 10 años
después de la muerte de su esposo debido a infarto agudo de miocardio. Signos vitales, con
temperatura de 37,2°C (98,9°F), pulso de 115/min, Presión arterial de 150/90 mmHg. Al
examen físico, paciente aparenta adolorida y está sudando profusamente. Actualmente está
respirando 100% de oxígeno por la máscara facial. No tiene distensión yugular. Sonidos
cardíacos distantes, pero regulares en ritmo y frecuencia. Un EKG no revela elevación del
segmento ST. Ella comenzó con aspirina, enoxaparina, atenolol, nitratos y morfina. ¿Cuál es el
siguiente paso en el manejo?
a) Adicionar atorvastatina
b) Adicionar clopidogrel
c) Adicionar enalapril
d) Intervención coronaria percutánea
e) Medición de troponina y CK-MB

Respuesta correcta: E
3. Femenina caucásica de 19 años que acude porque ha dejado de tomar en los últimos dos
meses sus pastillas anticonceptivas y una prueba de embarazo de orina resultó positiva. No
recuerda cuándo fue su último periodo. Se siente bien y no aqueja nada. Su historia médica
pasada sin significancia. Vive con su madre y dos hermanos. No fuma, no bebe o usa drogas
ilícitas. Su temperatura es de 37°C (98,6°F), Presión arterial de 104/78 mmHg, pulso de 72/min,
y Fr de 16/min. Examen físico revela sensibilidad mamaria, útero grávido, cérvix teñido de azul,
y es de otra manera sin complicaciones. USG muestra un embarazo intrauterino de 12
semanas. Laboratorios prenatales son analizados, y el frotis de PAP y orina y cultivo de cérvix
son obtenidos. Un análisis muestra bacteriuria moderada con esterasa leucocitaria y nitritos
positiva. Se le administró folato y vitaminas prenatales. ¿Cuál es el siguiente mejor paso en el
manejo?
a) No es necesario un manejo adicional como paciente asintomático
b) TMP/SMX
c) Nitrofurantoina
d) Levonogestrel
e) Míconazole

Respuesta correcta: C

4. Femenina de 45 años en sala de emergencias por aturdimiento después de un largo paseo


en carro. Su esposo reporta que no ha tomado fluidos, usualmente toma de 20-24 bebidas sin
alcohol diariamente, y orina frecuentemente. Presión arterial de 110/75 mmHg cuando esta
supina y 88/60 al sentarse, pulso de 80/min supina y de 102/min al sentarse. Laboratorios
muestran: Glucosa en orina negativo Osmolaridad urinaria 110 mosmol/Kg Osmolaridad
urinaria es de 176 después de administrar vasopresina. ¿Cuál de los siguientes es el
diagnóstico más probable?
a) Diabetes insípida central
b) Toma compulsiva de agua
c) Diabetes mellitus 1
d) Diabetes mellitus 2
e) Diabetes insípida Neurogénica
Respuesta correcta: A
5. Niño de 4 años es traído a la clínica pediátrica debido a debilidad y malestar. Él ha reportado
a su madre que estos síntomas se presentaron desde el último mes. Ella ha notado también
que cuando lo baña, lo siente inusualmente rígido. Él se niega a comer tanto como lo solía
hacer y no ha estado ganando peso. El también aqueja sensación de mareo y dolor en el
vientre. Sin antecedentes médicos relevantes. Tiene todas sus inmunizaciones al día. Al
examen físico, está afebril con Presión arterial de 80/48 mmHg. Es encontrado con hipotensión
ortostática. El resto del examen sin alteraciones. Laboratorios: Glóbulos blancos: 6,8/mm3
Plaquetas: 252,000/mm3 Sodio: 130 mEq/L Cloro: 97 mEq/L BUN: 14 mg/Dl Hematocrito: 46%
Glucosa: 70 mg/dL
Potasio: 5,6 mEq/L Bicarbonato: 24 mEq/L Creatinina: 0,6 mg/dL. Una prueba de estimulación
demuestra respuesta subóptima de ACTH. Un ensayo de ácidos grasos de cadena larga
demuestra altos niveles en la sangre del niño. ¿Cuál de los siguientes es característico de este
diagnóstico?
a) Trierucato de glicerol (aceite de Lorenzo) se ha demostrado que retrasa la progresión
de los síntomas una vez ellos han comenzado
b) Es un desorden ligada a X
c) Imagen de RM del cerebro es usualmente normal
d) Hallazgos neurológicos usualmente preceden al diagnóstico de insuficiencia adrenal
e) El gen responsable es una proteína de la membrana mitocondria.

Respuesta correcta: B

6. Chico de 17 años vienen por historia de fiebre de 10 días, malestar, dolor de garganta. El
tomo medicamentos de resfriado sin mejora de los síntomas. Tomo alguna ampicilina sobrante
que encontró en la cabina de medicamentos hace 7 días. Ha tenido sarpullido rojo en los
últimos 4 días. Es un chico muy activo en el deporte y ha tenido que perderse 2 juegos de
basketball en la pasada semana. Sin relevancia en historia médica. Es sexualmente activo y
usa condón consistentemente. Su temperatura es de 38,7°C (101,6°F). Al examinarlo presenta
amígdala eritematosa y edematosa con exudado claro. Presenta ganglios linfáticos
submandibulares agrandados y sensibles. Examen cutáneo muestra una erupción
maculopapular y eritematoso en el tronco y extremidades. El borde esplénico se palpa 2 cm por
debajo del margen costal. Una prueba rápida de estreptococo es negativa. ¿Cuál de los
siguientes es la causa más probable de la condición del paciente?
a) Estreptococo beta hemolítico de grupo A
b) Virus del Epstein Barr
c) Virus herpes simplex
d) VIH V
e) Rubivirus

Respuesta correcta: B

7. Niño de 5 años es visto en visita rutinaria de salud, y se encuentra en crecimiento adecuado.


Está cumpliendo con el desarrollo de todos los hitos y generalmente está saludable. Al examen
físico, tiene apariencia masculina saludable, con el borde hepático 3cm debajo del margen
costal. Laboratorios realizados con resultados: Glóbulos blancos 3,670/mm3 Hb 10,3g/dL
Hematocrito 36% Plaquetas 130,000/mm3. Una radiografía abdominal confirma aumento de
tamaño de hígado y bazo, y muestra osteopenia de cuerpos vertebrales. Una inspección de
seguimiento óseo muestra resplandecimiento de la porción tubular inferior de los fémures
distales. ¿Cuál sería la causa más probable de estos hallazgos?

a) Alfa-mannosidosis
b) Enfermedad de Gaucher (GD)
c) Síndrome de Hunter
d) Enfermedad de Niemann Pick tipo A
e) Enfermedad de Sanfíllipo tipo B

Respuesta correcta: B
8. Masculino de 24 años que viene para seguimiento después de completar curso de 6 meses
de tratamiento para tuberculosis activa que adquirió durante un viaje. Actualmente se siente
bien, y no especifica queja. Entre sus antecedentes médicos incluyen un nuevo caso de
tratamiento difícil de dermatitis seborreica y un episodio reciente de herpes zoster. Además de
esas condiciones, el paciente no presenta enfermedades crónicas. Signos vitales y examen
físico sin complicaciones, con la excepción de hallazgos cutáneos consistente con dermatitis
seborreica. El paciente está preocupado, sin embargo, acerca de sus posibilidades de ser un
caso recurrente de TBC. ¿Cuál de los siguientes es la prueba más apropiada para ofrecer al
paciente?
a) Prueba cutánea de alergia
b) Radiografía de tórax
c) Prueba de HIV
d) Colocación de PPD
e) Cultivo y frotis de esputo
Respuesta correcta: D

9. Masculino de 29 años de edad que se presenta al doctor debido a que ha notado que su ojo
se ha tornado amarillo. Esto se ha presentado intermitentemente desde hace 2 años. No
presenta ningún otro síntoma y se siente bien. Niega síntomas gripales recientes, malestar,
mialgias o dolor en cuadrante superior derecho. No ha tenido pérdida de peso, distensión
abdominal, hematemesis o hemorroides. Es heterosexual exclusivamente y ha tenido múltiples
parejas sexuales. Alguno de los encuentros sexuales han sido sin protección. Está actualmente
soltero. Toma alcohol y fuma socialmente. Su historia familiar es notable solamente por
hipertensión y enfermedad arterial coronaria. Niega uso de drogas intravenosas y nunca ha
recibido transfusión. Al examen físico, está bien desarrollado y nutrido. Está afebril y
normotenso. Examen de cabeza y cuello revelan ictericia escleral sin linfadenopatía. Examen
cutáneo no revela
angioma en araña o eritema palmar. No tiene ginecomastia. Examen cardiovascular y pulmonar
normal. Examen abdominal no revela distensión, cabeza de medusa o sensibilidad en
cuadrante superior derecho. Laboratorios: Creatinina: 0,8 mg/dL Glucosa: 104 mg/dL
Albúmina: 4,1 g/dL AST: 33 U/L ALT: 25 U/L Alcalina fosfatase: 65 U/L LDH: 110 U/L
Haptoglobina: 102 mg/dL Bilirrubina total: 3,2 mg/dL Bilirrubina conjugada 0,3 mg/dL
Anticuerpos de superficie de hepatitis B: Positivo IgM anti hepatitis A: Negativo Anticuerpos anti
hepatitis C: Negativo Frotis de sangre periférica: Sin células anormales. ¿Cuál de los siguientes
es la explicación más probable para la ictericia del paciente?
a) Infección aguda de hepatitis B
b) Síndrome de Gilbert
c) Hemólisis
d) Masa en la cabeza del páncreas
e) Cirrosis biliar primaria

Respuesta correcta: B

10. Femenina de 45 años es traída a sala de emergencias por su hermana 20 minutos


después de una convulsión generalizada. La hermana afirma que la paciente ha tenido
episodios intermitentes de sacudidas, palpitación, sudoración y ansiedad por 6 meses. No toma
medicamentos. Trabaja como farmacéutica. Esta diaforética y alerta. Su temperatura es de
36,7°C (98°F), Presión arterial de 130/85 mmHg, pulso de 110/min, anticuerpos contra insulina
negativa. Laboratorios que muestran: Glucosa sanguínea: 45 mg/dL Insulina: 7 µU/mL (VN <3
µU/mL) Péptido C: 1,5 nmol/L (VN <0,2 nmol/L) Proinsulina: 9 pmol/L (VN <5 pmol/L). ¿Cuál de
los siguientes es el diagnóstico más probable?

a) Síndrome de Zollinger Ellison


b) Administración facticia de insulina
c) Glucagonoma
d) Insulinoma
e) Feocromocitoma

Respuesta correcta:D

11. Una mujer de 24 años de edad llega al departamento de emergencia debido a dolor y visión
borrosa en el ojo izquierdo desde hace 2 días. Ha tenido dificultad para respirar desde hace 2
meses. Su temperatura es de 37.9°C, presión arterial de 116/78, y pulso de 90 cpm. El examen
físico muestra nódulos eritematosos y sensibilidad en la cara anterior de ambas piernas. Los
pulmones se encuentran claros a la auscultación. El examen con lámpara evidencia fotofobia
en ojo izquierdo, eritema alrededor de la córnea(circumcorneal) y precipitados blanquecinos en
la córnea. La presión intraocular es normal. Una radiografía de tórax muestra ensanchamiento
biliar bilateral. ¿Cuál de las siguientes características es más probable en esta condición?
a) Diarrea sanguinolenta y tenesmo
b) Cronicidad, curso no-remitente
c) Enfermedad de glándulas parótidas y lacrimales
d) Infecciones recurrentes con microorganismos encapsulados
e) Poliartritis simétrica

Respuesta correcta: E

12. Una mujer de 27 años de edad es admitida al hospital debido a dolor articular de 4 días de
evolución. Inicialmente tuvo un dolor articular difuso que “migró" desde sus muñecas hacia sus
codos y hacia sus rodillas. Durante las últimas 24 horas, sin embargo, ha tenido un intenso
dolor e hinchazón localizados en la rodilla derecha. Niega traumas en el área. Niega fiebre y
escalofríos, pero refiere leve disuria que atribuye a una infección de vías urinarias. Su
temperatura es de 38.1°C, presión arterial 120/70, pulso de 98 cpm, frecuencia respiratoria de
18 cpm. Al examen físico su rodilla está hinchada y sensible a la palpación. Tiene pústulas
necróticas y diseminadas en sus palmas y plantas. Se realiza una artrocentesis y el frotis Gram
muestra diplococos gram negativos. ¿Cuál de los siguientes es el siguiente paso más
apropiado para el manejo?
a) Ceftriaxona IV
b) Nafcilina IV
c) Penicilina IV
d) Vancomicina IV
e) Aguante los antibióticos y espere por el cultivo y resultados de sensibilidad

Respuesta correcta: A

13. Un hombre de 38 años con historia de herpes genital, acude al médico por dolor e
hinchazón de su rodilla derecha. Reporta que sus síntomas inician temprano en el día y le
resulta extremadamente doloroso el movimiento o tocarse la rodilla. Niega trauma en el área
afectada. Comenta haber tenido relaciones sexuales recientemente sin protección con una
prostituta. Su temperatura es de 39°C, presión arterial 125/90, pulso regular de 108 cpm y
frecuencia respiratoria de 15 cpm. El examen físico revela una rodilla derecha sensible a la
palpación e hinchada, con efusión palpable. El paciente se encuentra taquicárdico, pero por lo
demás al examen físico está normal. Se realiza una artrocentesis de la rodilla derecha y se
envía el fluido para su análisis. ¿Cuál de los siguientes hallazgos es más probable?
a) Conteo celular con 75 000 WBC X
b) Cultivo positivo para cocos Gram positivos en racimos
c) Frotis Gram que revela diplococos Gram positivos
d) Cristales con birrefigencia negativa
e) Cristales en forma de rombo

Respuesta correcta: A

14. Femenina de 34 años acude al médico por dolores mensuales por 12 meses. También ha
tenido movimientos intestinales dolorosos, sin sangre en heces. Ella ha sido incapaz de
concebir a pesar de tener relaciones sexuales regulares. Ha estado bajo uso de
medroxiprogesterona por 8 años, pero hace 15 meses que ha dejado de inyectarse, desde ese
tiempo ha tenido periodos regulares. Examen pélvico muestra una apariencia normal de vulva,
vagina y cérvix. Sin masas anexas. Examen rectovaginal muestra nodulaciones en ligamento
uterosacro con dolor a la palpación. Estudios de laboratorios muestran; Hb: 10.3g/dL Hct:
30.5% MCV: 74 microm3 Glóbulos blancos: 9,300 mm3 Neutrófilos: 56% Bandas: 3%
Plaquetas: 205,000 mm3 VES (ESR): 68 mm/h. ¿Cuál de los siguientes es el sitio anatómico
más común afectado por esta condición?
a) Vejiga
b) Ovarios
c) Peritoneo parietal
d) Órganos peritonéales
e) Recto

Respuesta correcta: B

15. Femenina de 83 años de edad que acude porque ella se “siente muy cansada todo el
tiempo”. Hematocrito en la oficina es 35%. Frotis de sangre periférica muestra algunos glóbulos
rojos macrocíticos. En la entrevista, la mujer cuyas finanzas están limitadas, admite que ha
estado bajo dieta estricta principalmente de pan. Ella ha estado tomando sustituto de jugo de
naranja en polvo. Ella no ha estado tomando vitaminas porque siente que no puede pagarlas.
¿Una deficiencia nutricional de cuál de los siguientes es la causa más probable de anemia en
este paciente?
a) Folato
b) Hierro
c) Vitamina B12
d) Vitamina C
e) Vitamina K

Respuesta correcta: A

16. Una mujer de 21 años, primigesta, con 39 semanas de gestación, viene a la sala de labor y
parto con contracciones dolorosas cada 3 minutos. Su evolución prenatal tiene datos
irrelevantes. Al examen muestra un cuello con 3 cm de dilatación y borramiento de 90%. La
FCF en un trazo es de 150 LPM y está reactivo. 5 horas después, al examinar el cuello, se
evidencia que tiene 9 cm y se encuentra en plano -1. Los trazos de la FCF muestran
desaceleraciones variables moderadas con cada contracción y variabilidad disminuida. Un
muestren de cuero cabelludo del feto es realizado y arroja los siguientes resultados, pH: 7.04,
7.05 y 7.06. ¿Cuál de los siguientes es el siguiente paso más apropiado en el manejo?
a) Cesárea
b) Episiotomía
c) Manejo conservador
d) Parto asistido con fórceps
e) Parto asistido con ventosas

Respuesta correcta:A

17. Un hombre de 61 años, es traído al cuarto de urgencias por dolor en el pecho. Él tiene una
historia de enfermedad coronaria de larga data e hipertensión y se le realizó una derivación
coronaria hace 6 años. El paciente tiene una angina estable crónica que es precipitada
usualmente por actividad y que se alivia con el reposo. Su medicación es aspirina, captopril y
metoprolol. Hace 3 semanas, su médico le prescribió sildenafil y ha usado el medicamento con
éxito. Su último encuentro sexual fue hace 4 días. Esta mañana, ha desarrollado un malestar
en el pecho. Sus signos vitales en el cuarto de urgencias son: Temperatura 36.1 °C, BP
220/120 mmHg, pulso en 100 por minuto y frecuencia respiratoria en 22 por minuto. Al examen
ocular muestra papiledema, el ECG revela cambios no específicos. ¿Cuál de las siguientes
opciones, es el mejor tratamiento indicado para este momento?
a) Labetalol intravenoso
b) Morfina intravenosa
c) Clonidina oral
d) Metoprolol oral
e) Nitroglicerina oral

Respuesta correcta:A

18. Una mujer de 29 años, saludable, estaba subiendo las escaleras en su nueva casa
hace un mes, cuando la casa, colapsó. Ella fue traída al hospital con una fractura de fémur
izquierdo. Se le hizo una interconsulta al servicio de psiquiatría porque la paciente se quejaba
de pesadillas y recuerdos y tenía miedo de ir a dormir por esto. Durante la entrevista, ella tenía
llanto fácil y estaba asustada por su miedo a caer y esto le impide participar plenamente en su
rehabilitación, por lo cual, ella teme que los médicos le den salida. ¿Cuál de los siguientes es el
tratamiento más apropiado?
a) Psicoterapia orientada introspectivamente
b) No dar terapias porque la paciente necesita tomar responsabilidad para su tratamiento
c) Colocar a un cuidador que esté con la paciente en la sala, las 24 horas para calmar su
ansiedad
d) Iniciar antidepresivos
e) Iniciar benzodiacepinas

Respuesta correcta: D

19. Un hombre de 43 años acude al médico por historia de 4 años con dolor articular. Tiene
dolor en las pequeñas articulaciones proximales y distales de su mano derecha, en la rodilla
izquierda, el tobillo derecho y el codo izquierdo. Ha tenido rigidez matutina que dura hasta 2
horas. Su historial médico es significativo solo por “unos parches escamosos en mis codos”
que nunca han sido atendidos por un médico. Su temperatura es de 37°C, presión arterial de
120/70, pulso 76 cpm y frecuencia respiratoria de 13 cpm. El examen físico muestra
deformidades en las articulaciones interfalángicas proximales y distales (subluxaciones) del
dedo índice, medio y anular de la mano derecha; deformidad interfalángica proximal del dedo
anular de la mano izquierda; todos los dedos de las manos presentan punteado ungueal (signo
de la uña en dedal) y decoloración de las uñas. Laboratorios: Hb-14.0g/Dl Hematocrito: 42%
Glóbulos blancos en 5 000/mm3 Plaquetas: 200.000/mm3 Factor Reumatoide Negativo y VES:
46 mm/h ¿Cuál es el diagnóstico más probable?
a) Onicomicosis
b) Osteroartritis
c) Pseudogota
d) Ártriris psoriásica
e) Artriris Reactiva

Respuesta correcta:D

20. Una mujer de 65 años con cáncer metastásico es admitida al hospital por un severo dolor
en su pierna izquierda desde hace 2 semanas. Él está en su muslo izquierdo cerca de su
cadera. Ha tenido problemas para caminar debido al dolor. El dolor empeora en la noche. No
tiene fiebre, rigidez matinal, o trauma. La fosfatasa alcalina sérica de la paciente se encuentra
en 400 U/L. Una radiografía del muslo muestra una lesión lítica en su fémur proximal izquierdo.
¿Cuál de las siguientes es más probable que sea la malignidad de la paciente?
a) Mama
b) Colon
c) Melanocítico
d) Páncreas
e) Tiroides

Respuesta correcta:D

21. Un hombre de 63 años acude al médico para que le evalúe su dificultad para caminar
desde hace 6 meses. Refiere debilidad en la parte inferior de las piernas que mejora con el
ejercicio y a lo largo del día. Hace tres meses, empezó a añadir semillas de lino a su desayuno
porque sufre estreñimiento con frecuencia. Tiene hipertensión y diabetes mellitus de tipo 2. Ha
fumado 2 paquetes de cigarrillos diarios durante los últimos 45 años. Su medicación actual
incluye enalapril y metformina. Las constantes vitales están dentro de los límites normales. La
exploración muestra sequedad de las mucosas. La fuerza muscular de las extremidades
inferiores está disminuida. La sensibilidad al pinchazo y al tacto ligero es normal. Los reflejos
tendinosos profundos son 1+ bilateralmente. La contracción muscular activa o los golpes
musculares repetidos aumentan la actividad refleja. ¿Cuál de las siguientes opciones es más
probable que confirme el diagnóstico?
a) Fasciculaciones y ondas picudas en Electromiografía
b) Autoanticuerpos contra canales de Ca voltaje dependientes
c) Infiltrados endomisiales en biopsia de músculo
d) Placas desmielinizantes en una RMN de SNC
Respuesta correcta B
22. Una mujer de 39 años, con antecedentes de migrañas, acude a urgencias por una cefalea
intensa, repentina y punzante y por visión doble durante una hora. Dice que ha tenido dolores
de cabeza frecuentes y que no ha tenido la menstruación en varios meses. Su presión arterial
es de 93/61 mm Hg. Las pruebas de campo visual muestran una disminución de la agudeza
visual y pérdida de la visión periférica en ambos ojos. ¿Cuál de los siguientes es el diagnóstico
más probable?

A) Isquemia cerebral transitoria


B) Apoplexia pituitaria
C) Síndrome de Sheehan
D) Migraña con aura
E) Hemorragia subaracnoidea
F) Cefalea en racimos

Respuesta correcta : B

23. Un hombre de 50 años acude al médico por dolor articular de 3 meses de duración. El dolor
va y viene y típicamente ocurre en una articulación, se resuelve, y luego se desarrolla en otra
articulación. El dolor no empeora ni mejora por la actividad. Niega rigidez matinal. Refiere
historia de una erupción en su hombro que apareció inmediatamente después de una de sus
excursiones de cacería, y resolvió hace 6 semanas. El paciente trabaja como oficial de policía,
y va de cacería 2 veces al mes. Bebe 1 o 2 cervezas ocasionalmente. Su temperatura es de
37.8°C y su pulso de 75 cpm. ¿Cuál de los siguientes microorganismos es la causa más
probable de sus síntomas?
a) Borrelia burgdorferi
b) Borrelia recurrentis
c) Staphylococcus aureus
d) Treponema carateum
e) Treponema pallidum

Respuesta correcta:A

24. Un niño de 4 años con una hidrocefalia congénita es traído al cuarto de urgencias con
fiebre, dolor de cabeza, irritabilidad, fotofobia y vómitos de 2 días de evolución. Él fue operado
hace un año para una derivación ventrículo-peritoneal y sus vacunas están al día. Su
temperatura es de 39.6 “C. Al examen físico, tiene rigidez nucal y signos de Kernig y
Brudzinski presentes. La piel que rodea el trayecto del catéter de derivación, está eritematosa.
Una punción lumbar y posterior análisis del LCR, muestra un conteo de leucocitos de 40 000 x
mm3 con 85% de neutrófilos. Glucosa en 48 mg/dl y proteínas en 169 mg/dl. ¿Cuál de las
siguientes opciones, es la causa más probable de la condición actual del paciente?
a) Haemophilus influenzae
b) Neisseria meningitidis
c) Pseudomonas aeruginosa
d) Staphylococcus epidermidis
e) Streptococcus pneumoniae

Respuesta correcta: D

25. Un hombre de 63 años viene al médico, después de que un farmacéutico le dice que “su
medicación es incorrecta”. Él tiene una historia de cirrosis hepática e hipertensión portal,
causada por abuso del alcohol. Él nunca ha tenido historia de sangrado gastrointestinal y
recientemente se jubiló de su trabajo como piloto comercial en una aerolínea. Él fuma un
paquete de cigarrillos todos los días y actualmente, no toma alcohol. ¿La administración
conjunta de los siguientes fármacos, es probable que sea peligroso?
a) Lactulosa y potasio oral
b) Prednisona y salbutamol inhalado
c) Propranolol y dinitrato de isosorbide
d) Espironolactona e hidroclorotiazida
e) Espirolactona y potasio oral

Respuesta correcta: E
26. Una mujer de 52 años quien está siendo tratada con quimioterapia después de una cirugía
por cáncer de mama, es traída al cuarto de urgencias. Ella tomó un antiemético ayer para las
náuseas y fue traída por su esposo porque ha tenido un comportamiento bizarro en las últimas
24 horas, con flexibilidad cérea en sus movimientos y mutismo. Ella no tiene historia
psiquiátrica previa y no tiene otras condiciones médicas, adicional al cáncer de mama. El
examen físico, los laboratorios y signos vitales, se encuentran todos normales. ¿Cuál de las
siguientes opciones, es la farmacoterapia más apropiada para el manejo?

a) Alprazolam
b) Benztropina
c) Haloperidol
d) Metilfenidato
e) Ácido valproico

Respuesta correcta: B

27. Una mujer de 24 años viene a su consultorio para su evaluación, posterior a la visita que
ella hizo hace dos días al cuarto de urgencias por una “reacción alérgica” que consiste en una
picazón en la garganta después de comer helado, seguido de náuseas y vómitos, una
sensación de rubor y urticaria. Este mismo tipo de problema le pasó hace un año, después de
comerse un dulce de chocolate y una vez antes, en un restaurante de comida tailandesa. Su
amigo sugirió que esto puede ser una alergia al maní, pero la paciente no recuerda si ha
comido maní cuando le han ocurrido esas reacciones. En este momento, todas las
manifestaciones de la reacción alérgica se han resuelto y ella luce saludable y no tiene una
historia significativa por enfermedades previas, su único medicamento actualmente, son
anticonceptivos orales. Al examen físico en el consultorio, todo está normal. ¿Cuál de las
siguientes opciones es el siguiente paso más apropiado en la evaluación y el manejo de esta
paciente?
a) Iniciar desensibilización (inyecciones de alérgeno) con extracto de maní
b) Decirle a la paciente que haga un registro de los alimentos que consume hasta el
próximo episodio alérgico
c) Iniciar un desafío con maní
d) Ordenar un prick test (prueba de pinchazo) en la piel o una prueba de
radioalergosorbencia para alergia al maní
e) Iniciar terapia profiláctica con bloqueadores H2 y difenhidramina

Respuesta correcta:D

28. Masculino de 50 años que acude por examen rutinario de mantenimiento de la salud.
Historia médica sin complicaciones, no toma medicamentos. Ha fumado 2 paquetes de
cigarrillos diarios por dos años, pero dejó de fumar hace 27 años. Su hermano murió de cáncer
pulmonar de células escamosas a los 56 años. Presión arterial de 122/84 mmHg, pulso de 74/
min, Fr: 12/min. Examen físico sin anormalidades. Está muy ansioso por la muerte reciente de
su hermano causado por el cáncer. ¿Cuál es el mayor riesgo que puede padecer el paciente de
las siguientes malignidades?

a) Carcinoma colorectal
b) Carcinoma hepatocelular
c) Carcinoma pulmonar
d) Carcinoma pancreático
e) Carcinoma prostático

Respuesta correcta:E

29. Un niño de 1 mes es llevado al médico por una historia de 1 semana de mala alimentación.
Desde que nació, ha mojado de 8 a 10 pañales al día y ha defecado de 3 a 4 veces por
semana. Su madre dice que defecó por primera vez a los 2 días de nacer. Nació a término
mediante un parto vaginal espontáneo en casa. Se alimenta exclusivamente con leche materna
y aún no ha recibido ninguna vacuna de rutina. Su madre ha emigrado recientemente desde
Mongolia y no ha acudido al médico en 10 años. No hay antecedentes familiares de
enfermedades graves. Parece aletargado. Está en el percentil 5 de longitud y en el percentil 20
de peso. Su temperatura es de 34,8°C, su pulso es de 92/min, su respiración es de 38/min y su
presión arterial es de 96/58 mm Hg. La exploración muestra ictericia y disminución del tono
muscular en las extremidades. El abdomen está distendido. El área perianal parece normal y el
examen rectal revela heces inspiradas. Los estudios de laboratorio muestran:

Hemoglobina 9,1 g/dL


Volumen corpuscular medio 110 µm3
Recuento de leucocitos 12.800/mm3 con un diferencial normal (N = 8000-16.000)
Recuento de reticulocitos 1%.
Suero
Na+ 137 mEq/L
K+ 3,7 mEq/L
Cl- 102 mEq/L
Ca2+ 8,7 mg/dL
Glucosa 95 mg/dL
Creatinina 0,8 mg/dL
Bilirrubina total 8,5 mg/dL
Directa 0,9 mg/dL
Fosfatasa alcalina 70 U/L
Aspartato aminotransferasa (AST, GOT) 35 U/L
Alanina aminotransferasa (ALT, GPT) 30 U/L
Proteína C reactiva 0,20 mg/L
Un frotis de sangre periférica muestra macrocitos. El análisis de orina y la ecografía del
abdomen no muestran anomalías. ¿Cuál de los siguientes pasos es el más apropiado para el
diagnóstico?

A. Bario por enema


B. Pruebas de función tiroidea
C. Biopsia de hígado
D. Cloruro en sudor
E. Análisis de heces

Respuesta correcta: B (Pruebas de función tiroidea)

30. Un hombre de 48 años acude al médico por pérdida de peso, fatiga y una erupción de 1
año de evolución. Niega náuseas/vómito, disfagia, hematoquezia, melena, diarrea o
constipación. Ha tenido tos productiva de esputo teñido con sangre y también sangre en la
orina. No fuma. Recientemente visitó un área rural de India hace 2 semanas. Tiene historial de
asma bronquial de larga data, Su medicación actual consiste en albuterol inhalado para la falta
de aire. Su temperatura es de 37.8°C, presión arterial 122/68, pulso de 89 cpm y frecuencia
respiratoria de 15 cpm. Su saturación de oxígeno es de 98% a aire ambiente. El examen físico
muestra dolor a la percusión de los senos paranasales. Las membranas mucosas están secas.
A la auscultación se escuchan sibilancias dispersas y sonidos respiratorios gruesos. Examen
cardíaco con SI y S2 normales, sin soplos audibles. No hay sensibilidad a la palpación
abdominal. Se observa una erupción elevada, palpable y papular que no blanquea con la
presión en la parte anterior de sus piernas. La fuerza muscular es 3/5 en dorsiflexión del pie
izquierdo y 2/5 para la extensión de la rodilla derecha. No hay signo de Banbinski. Una TC de
tórax muestra engrosamiento bronquial. Laboratorios: glóbulos blancos =12,100/mm3.
Linfocitos 8% Eosinófilos 10% Plaquetas: 490,000/mm3 CI-: l00mEq/L Creatinina: 2.2 mg/dl
ALT=31 U/L, Fosfatasa alcalina: 90 U/L Na+: 34 mEq/L, HCO3-: 20 mEq/L Glucosa: 103 mg/dl
C3: 90 mg/dl C4: 20 mg/dl Neutrófilos 75%, Hematocrito: 31% K+: 5.3 mEq/L, Nitrógeno de
urea: 35 mg/dl AST: 25 U/L, Anticuerpo Antinuclear: positivo 1:128 Anticuerpos Anti-citoplasma
de neutrófilo (ANCA): positivo 1:256 PPD: No reactivo. Urinálisis: cilindros hemáticos ¿Cuál es
el diagnóstico más probable?
a) Enfermedad de Churg-Strauss
b) Apergilosis diseminada
c) Tuberculosis diseminada
d) Carcinoma oculto
e) Granulomatosis de Wegener
Respuesta correcta: A

31. Un hombre de 50 años acude al médico por un dolor cerca de sus orejas de una semana
de evolución. Hace 3 días sus orejas se comenzaron a hinchar. Niega cualquier cambio en la
dieta o viajes recientes. Su historial médico es irrelevante. Toma multivitaminas diariamente. Su
temperatura es de36.8'’C, presión arterial de 110/80 y pulso de 75 cpm. Su saturación de
oxígeno es de 94% a aire ambiente. El examen físico muestra sensibilidad a la palpación e
hinchazón sobre las porciones cartilaginosas de ambas orejas. Las áreas no cartilaginosas no
muestran anormalidad. Una biopsia de la oreja muestra tejido de granulación en el cartílago.
¿Cuál de las siguientes pruebas de función pulmonar es más probable que presente este
paciente?
a) Disminución de la difusión de monóxido de carbono
b) Disminución es flujo espiratorio
c) Incremento en FEV1
d) Reducción de la capacidad residual funcional
e) Reducción de la capacidad vital

Respuesta correcta: B

32. Un albañil de 27 años llega a emergencias con su rodilla izquierda hinchada. El dolor
empezó hace 36 horas y le ha limitado su capacidad para trabajar. Ha tenido fiebre y
escalofríos desde las últimas 24 horas. El examen físico revela una temperatura de 38.9ºC y
pulso de 104 cpm. El resto del examen físico solo es importante para una rodilla izquierda
hinchada, eritematosa y con evidente efusión. El rango de movimiento se encuentra limitado.
La artrocentesis revela 90.000/mm3 leucocitos con 82% de neutrófilos. Un frotis Gram revela
muchos neutrófilos, pero no se visualizan microorganismos. La microscopía de luz polarizada
no revela cristales. Basado en la etiología más probable, cuál de las siguientes es el
tratamiento inicial más apropiado:
a) Vancomicina
b) Colchicina
c) Ceftriaxona
d) Nafcilina
e) Ceftriaxona+vancomicina

Respuesta correcta: E

33. Una mujer de 74 años acude al médico por historia de 3 meses de dolor óseo. El dolor no
se alivia con ibuprofeno. Tiene historia de hipertensión y enfermedad coronaria. Sus
medicamentos incluyen lisinopril, metoprolol y aspirina. Ha fumado 2 paquetes de cigarrillos
diarios por 30 años. No consume bebidas alcohólicas. Su temperatura es de 36.8°C, presión
arterial 130/85 y pulso de 65 cpm. El examen físico muestra dedos en palillo de tambor (dedos
hipocráticos) en ambas manos. A la auscultación se escucha una fase espiratoria prolongada y
un galope S4. Hay sensibilidad a la palpación de las porciones distales de sus huesos.
Radiografías de antebrazos y fémur revelan periostosis bilateral. ¿Cuál de las siguientes
opciones es la más adecuada para el siguiente paso en el manejo?
a) Biopsia de médula ósea
b) Radiografía de tórax
c) Incrementar la dosis y frecuencia del ibuprofeno
d) Aspirado articular
e) No está indicado manejo adicional

Respuesta correcta: B

34. Una mujer de 60 años se queja de incremento de fatiga desde hace unos meses
atrás. Reporta que sus actividades diarias regulares le producen dolor y rigidez,
especialmente lavarse el cabello. También ha tenido problemas para encontrar una posición
cómoda para descansar. Ha perdido 12 libras de peso corporal en los últimos 3 meses, pero
antes de esto se sentía saludable y no está bajo ninguna medicación. El examen físico es
normal. ¿Cuál de los siguientes es el diagnóstico más probable?
a) Depresión
b) Hipokalemia
c) Polimialgia reumática
d) Polimiositis
e) Arteritis de la temporal

Respuesta correcta:C

35. Una mujer de 26 años acude al médico por historia de 3 meses de dolor en las
articulaciones y en las manos. Refiere mínima mejora con ibuprofeno; el dolor ha estado
interfiriendo con su trabajo como arquitecta. Ella ha tenido una erupción enrojecida después de
hacer deportes en exteriores. Ha tenido dolor en la boca mientras cepilla sus dientes. Su
temperatura es de 37.4°C, presión arterial de 130/90, pulso de 76 cpm. El examen físico
muestra hinchazón en la articulación interfalángica proximal del dedo índice de la mano
izquierda. Tiene múltiples úlceras puntiformes en la mucosa oral.
Laboratorios: Na+: 141 mmol/L HCO3-: 23mmol/L Leucos: 4,700/mm3 Eosinófilos: 1% K+: 5.0
mmol/L CI-; 97 mmol/L Hb: 12.5 mg/dl Plaquetas: 20,000/mm3 Neutrófilos segmentados:
55% Bandas: 3% Basófilos: 0.50% Linfocitos: 33% Monocitos: 3% ANA sérico positivo y Anti-
dsDNA positivo ¿Cuál de las siguientes opciones es la más
probable como indicación para el uso de esteroides en este paciente?
a) Anemia hemolítica
b) Dolor articular refractario a ibuprofeno
c) Trombocitopenia
d) ANA sérico positivo
e) Pericarditis

Respuesta correcta: C

36. Una mujer de 56 años acude al médico por un severo dolor de espalda y dolor de
cadera moderado desde hace 2 días. Los síntomas empezaron cuando intentó levantar a su
gato. Ha tenido fatiga y malestar general desde hace 3 meses. Niega disfunción neurológica o
trauma. SU historial médico es irrelevante. Su presión arterial es de 135/85 y pulso de 80 cpm.
Al examen físico presenta extrema sensibilidad a la palpación en la columna media lumbar e
incapacidad para flexionar y extender la espalda debido al dolor. Las radiografías de la columna
lumbar y pelvis muestran una fractura por compresión de la vértebra L3; también hay varias
áreas de radiolucencias redondas en su pelvis y fémur proximal. Los laboratorios muestran lo
siguiente: Hb: 11g/dl VCM=85fl Glóbulos blancos: 8,000/mm3; Plaquetas=200,000/mm3
Proteínas totales: 9.8 mg/dl Albúmina: 3.0mg/dl Ca2+:12.0mg/dl Fosfatasa alcalina: 65U/L
VES: 35mm/hr ¿Cuál de los siguientes es el diagnóstico más probable?
a) Herniación de disco lumbar
b) Cáncer metastásico a hueso
c) Mieloma Múltiple
d) Osteoporosis
e) Enfermedad de Paget ósea

Respuesta correcta: C

37. Un hombre de 56 años es admitido al hospital porque tiene dificultad respiratoria. Se le


coloca un tubo endotraqueal para darle ventilación mecánica con un volumen corriente de 900
cc con una frecuencia respiratoria de 12 por minuto y una FIO2 de 50%. La presión positiva al
final de la espiración es de 10 cmH2O. Sus medicamentos actuales incluyen heparina
subcutánea y aspirina. Dos días después, él desarrolla taquicardia y tiene una presión arterial
de 110/70 mmHg. Al examen cardíaco se encuentran contracciones prematuras múltiples. Una
gasometría muestra una pO2 de 40 mmHg. ¿Cuál de las siguientes opciones es la causa más
probable de la condición del paciente?
a) Secreciones bronquiales
b) Arritmia cardíaca
c) Infarto al miocardio
d) Neumotórax
e) Tromboembolismo pulmonar

Respuesta correcta: D

38. Un estudiante de medicina de 22 años viene a la clínica quejándose de debilidad


severa, dolor de garganta y fiebre de 38.9 °C desde hace 4 días, adicional un exantema
generalizado de color rosado durante los últimos dos días. Ha tenido buena salud antes de este
episodio y después de obtener una historia clínica y de realizar el examen físico, se ha
constatado que el paciente no ha tenido contacto con personas enfermas, él tiene una relación
monogámica heterosexual desde hace un año y no ha viajado en el último año. El niega el uso
de drogas ilícitas y admite que se ha automedicado con amoxicilina Para su “faringitis
estreptocócica”. Él dice que “yo ahora he desarrollado fiebre escarlatina. Doctor, ayúdeme”. El
aparenta estar pálido y exhausto. El examen físico muestra una linfadenopatía difusa e
inflamada y sus amígdalas están eritematosas. Se le palpa la punta del bazo y el borde
hepático y tienen un ligero dolor a la palpación. Un hemograma completo se le realiza en el
momento y revela lo siguiente: Hematocrito: 42% Hemoglobina: 14 g/dl Plaquetas: 140,000 x
mm3 Leucocitos: 4500 x mm3 Neutrófilos: 35% Linfocitos: 60% Eosinófilos: 2% Monocitos: 3%.
Se le realizó un frotis sanguíneo en el que se encontraron >10% de linfocitos con abundante
citoplasma. ¿Cuál de las siguientes opciones, es el siguiente paso para confirmar el
diagnóstico?
a) Aminotransferasas hepáticas y bilirrubina
b) Prueba de anticuerpos heterófilos
c) Prueba de IgG para el antígeno de la cápside del virus de Epstein Barr
d) lgM para citomegalovirus
e) IgM para Toxoplasma

Respuesta correcta:B

39. Un hombre de 24 años viene al médico porque tiene una historia, desde hace una
semana de fiebre, tos productiva con sangre, sibilancias y dificultad para respirar. Él ha
desarrollado un exantema pruriginoso en el pecho y el abdomen desde ayer. Es granjero en
una zona rural del sureste de los Estados Unidos. Tiene una frecuencia respiratoria de 30 por
minuto y a la auscultación presenta sibilancias, adicional presenta un exantema urticariforme
en el tronco. Al examen abdominal tiene una distensión leve y malestar difuso a la palpación.
Se le realiza una radiografía de tórax y muestra varios infiltrados en forma de parche,
bilaterales, pero tiene más en el campo pulmonar derecho. En los laboratorios se evidencian:
Hemoglobina: 14.3 g/dl Leucos: 5 400 x mm3 Eosinófilos: 30% Neutrófilos segmentados: 35%
Linfocitos: 30% Monocitos: 5% ¿Cuál de los siguientes, es la farmacoterapia más apropiada
para el tratamiento?
a) Ivermectina
b) Levamisol
c) Mebendazol
d) Pamoato de pirantel
e) Tialbendazol

Respuesta correcta:C

40. Femenina de 39 años primigrávida de 40 semanas de gestación acude a sala de parto con
contracciones, sus controles prenatales fueron sin complicaciones. Ella tiene síntomas de
hipotiroidismo para la cual toma levotiroxina. No tiene alergias a medicamentos conocidos. Sus
laboratorios prenatales muestran: Tipo de sangre: A negativo Antígeno de superficie de
hepatitis B: Negativo Rubéola: Inmune Reagina rápida en plasma: No reactivo HIV: Negativo
Prueba de carga de glucosa: l00mg/dL. La paciente da a luz a una niña de 6 libras, 14 onzas
con puntaje de APGAR 9/9. El tipo sanguíneo de la niña es A positivo. El plan de la paciente es
alimentar al infante con lactancia materna exclusiva. ¿Cuál de los siguientes es el paso más
apropiado en el manejo?
a) Administrar inmunoglobuina antihepatitis B y vacuna de la Hepatitis B
b) Administrar RhoGAM
c) Administrar inmunización para rubéola
d) Empezar metroclopramida
e) Comenzar píldoras de anticonceptivos orales
Respuesta correcta:B

41. Una bebé de 8 días de edad es traída al médico por sus padres debido a una decoloración
cutánea amarillenta. Ella nació a las 39 semanas de gestación y ha sido alimentado con pecho
desde entonces. Examen físico muestra ictericia significativa del tronco y cara. Sin ictericia
escleral. Por otra parte se encuentra saludable. Una revisión de su historial muestra que tanto
su madre y ella son Rh-positivo. ¿Cuál de los siguientes es la causa más probable de la
condición del infante?
a) Ictericia por leche materna
b) Deficiencia de glucosa 6 fosfato deshidrogenasa
c) Ictericia por falla en alimentación de leche materna
d) Ictericia fisiológica
e) Incompatibilidad por Rh

Respuesta correcta:A

42. Femenina de 38 años grava 3 para 2 a la décima semana de gestación que acude para
control prenatal. Se siente muy bien solo con náuseas ocasionales. Sus historias obstétricas
fueron de dos partos vaginales espontáneos a término sin complicaciones de embarazo. Tiene
como antecedentes médico significativo la hipertensión y ha estado en medicación de vez en
cuando, sin embargo no recuerda el nombre del medicamento. Ella nunca ha tenido una cirugía
y no ha tenido alergias a medicamentos. Su Presión arterial inicial es de 144/92 mmHg. Al
Examen físico fue normal para una mujer en su 10° semana de gestación. Se le repite la
Presión arterial y es de 140/90mmHg. 2 semanas después ella regresa para toma de Presión
arterial la cual tuvo en 142/92mmHg. ¿Cuál de los siguientes es el próximo paso más
apropiado en el manejo?
a) Continuar con seguimiento de la Presión arterial
b) Empezar con alfa metildopa
c) Empezar con atenolol
d) Empezar con captopril
e) Empezar con labetalol

Respuesta correcta:A

43. Un hombre de 66 años acude al médico por debilidad progresiva desde hace 3
meses. El paciente ya no es capaz de subir escaleras. El paciente presenta una erupción en la
parte superior del tórax y en la cara. Niega hormigueo, adormecimiento, cefalea, dificultad para
tratar o incontinencia. No toma medicamentos. Su temperatura es de 37.2°C, presión arterial de
135/80 y pulso de 80 cpm. El examen físico muestra lesiones eritemato-escamosas en las
áreas ciliares, laterales al tabique nasal y párpados, en ambos lados de la cara. Se evidencia
debilidad bilateral de la musculatura proximal; reflejos osteotendinosos disminuidos.
Laboratorios se muestran a continuación Na+: 138 mEq/L CI-: 102 mEq/L K+: 4.4 mEq/L
HC03-: 25mEq/L Creatina Kinasa: 30,000 ng/MI Anticuerpo antinuclear: Positivo ¿Cuál de las
siguientes opciones es el siguiente paso más apropiado en el manejo?
a) Prednisona
b) Electromiograma
c) Immunoglobulina IV
d) Biopsia de músculo
e) Plasmaferesis

Respuesta correcta:B

44. Una mujer de 23 años acude al médico por dolor en el lado derecho del tórax que empeora
a la inspiración. Refiere que ha tenido una erupción pruriginosa en la cara e hinchazón
dolorosa en sus dedos de las manos. No toma medicamentos. Su temperatura es de 37°C,
presión arterial de 160/90, pulso de 98 cpm y frecuencia respiratoria de 15 cpm. El examen
físico muestra lesiones papuloeritematosas en ambas mejillas y el dorso de la nariz. La
paciente presenta hinchazón de las articulaciones interfalángicas proximales y
metacarpofalángicas bilateralmente. Los ruidos respiratorios están disminuidos en el campo
pulmonar inferior derecho. La hemoglobina es de 8g/dL. ¿Cuál de las siguientes pruebas
diagnósticas es más probable que se encuentra anormal?
a) Anticuerpos anti-centromero
b) Anticuerpos anti-DNA de doble cadena
c) Anticuerpos anti-histonas
d) Anticuerpos anti-Jo-1
e) Anticuerpos anti-nucleares

Respuesta correcta:E

45. Una mujer de 32 años viene al médico para una evaluación de rutina. Ella dice que ha
tenido palpitaciones episódicas, pero niega que ha tenido dolor en el pecho, falta de aliento u
otros síntomas significativos. No ha tenido historia de enfermedades relevantes y no toma
medicamentos. Su temperatura es de 36.4 °C, Presión arterial de 120/70 mmHg, pulso en 72 x
minuto, respiratorio 16x minuto. Peso de 65 kg, altura de 170 cm y IMC de 22.5 kg/m2. Al
examen físico no se encuentran anormalidades y los laboratorios están normales. Un ECG
muestra contracciones atriales prematuras ocasionales y un ecocardiograma, muestra una FE
normal y no presenta anomalías valvulares. ¿Cuál de las siguientes opciones, es el mejor
siguiente paso en el cuidado de la paciente?
a) Digoxina
b) Diltiazem
c) Tranquilizar a la paciente
d) Flecainida
e) Metoprolol

Respuesta correcta:C

46. Un hombre de 50 años que tiene una historia de abuso del alcohol durante los últimos 20
años, es traído al hospital, quejándose de fiebre, mareos y tos. Niega otros antecedentes.
Tiene tos productiva con esputo de aspecto marrón y niega otros síntomas, no toma
medicamentos y es alérgico a las sulfas. Tiene una temperatura de 37.5 °C, Presión arterial de
110/70 mmHg, pulso de 91 cpm y respiración de 14 rpm Al examen físico se evidencia crépitos
en el lóbulo pulmonar superior izquierdo y al examinar el corazón, tiene ritmo y frecuencia
regular. Una radiografía de tórax muestra que tiene un infiltrado en el lóbulo superior del
pulmón izquierdo. ¿Cuál sería el manejo farmacológico más apropiado?
a) Azitromicina
b) Cefuroxima y azitromicina
c) Clindamicina
d) Éritromicina y levofloxacina
e) Vancomicina

Respuesta correcta: B

47. Una mujer de 34 años HIV positiva viene al médico con su pareja porque tiene una historia
de malestar y artralgias desde hace 4 días. Su pareja dice que la paciente ha tenido episodios
de confusión y sensación de hormigueo en sus brazos y piernas desde hace 2 horas, su pareja
está preocupada porque la paciente usó heroína hace 3 días y todavía está sangrando en el
sitio donde se inyectó. Sus medicamentos son zidovudina, didanosina, indinavir y trimetoprim
con sulfametoxazol. La paciente está orientada en persona, pero no en tiempo y lugar. Su
temperatura es de 39.1, PA 150/90 mmHg, pulso en 110 x’ y la FR en 20 x’. Al examen físico se
evidencian escleras ictéricas y tiene petequias no palpables en el pecho y en las extremidades,
sus pulmones están claros a la auscultación y a la palpación abdominal, tiene sensibilidad
difusa y leve a la palpación. La punta del bazo se le palpa a 4 cm por debajo del reborde costal.
Los laboratorios muestran lo siguiente; Hemoglobina: 7.1 g/dl Plaquetas: 50 000 x mm3
CD4: 230 x uL Leucos: 4 600 x mm3 Tiempo de sangría: 11 minutos (N: 2-7 min) Frotis de
sangre periférica: Glóbulos rojos fragmentados Proteínas en orina: 500 mg/dl Glóbulos rojos en
orina: 50 x campo ¿Cuál de los siguientes hallazgos serían los más probables en esta
paciente?

a) Niveles de creatinina normales


b) Niveles de bilirrubina indirecta normales
c) Niveles de lactato deshidrogenase normales
d) Tiempos de protrombina y de tromboplastina parcial activada normales
e) Conteo de reticulocitos normales

Respuesta correcta:D

48. Femenina de 60 años con historia de osteoartritis acude al médico debido a un dolor en su
rodilla izquierda. Su dolor no se alivia con 2,2 mg de ibuprofeno por día. Ella no presenta dolor
abdominal ni sangre en heces. Ella ha ganado 4,5 Kg (10 Ib) de peso durante el último año.
Ella pesa 90,7 Kg y talla 160 cm (63in). Su temperatura es 37°C (98,6°F), Presión arterial de
125/75 mmHg, pulso de 70/min. Tiene una efusión articular de tamaño mediano en la rodilla
izquierda. Hay una sensibilidad difusa a la palpación, crépitos e inestabilidad articular leve tanto
medial como lateral. ¿Cuál de los siguientes es el próximo paso más apropiado en el manejo?
a) Adicionar acetaminofén a su régimen
b) Incrementar dosis de ibuprofeno
c) Inyección intraarticular de esteroides, cada 2 semanas por 6 meses
d) Reemplazo de rodilla
e) Pérdida de peso más ejercicio de resistencia

Respuesta correcta:E

49. Masculino de 66 años es traído a sala de emergencias quejándose de fatiga, náusea


vómito y pobre apetito por una semana. Tiene historia de HTA de larga data, bien controlada
con lisinoprol, 40mg/día y nifedipina, 90mg/día, diabético y sufre de insuficiencia renal crónica.
Ha comenzado con diálisis hace 1 año pero no ha sido capaz de ir en las últimas dos semanas
debido a falta de transporte. Su temperatura es de 37°C (98,6°F), presión arterial de 186/82
mmHg, pulso de 90/min y frecuencia respiratoria de 20/min al examen físico muestra distensión
venosa yugular en 5cm y crépitos bibasales en los pulmones, y al examen cardiaco revela un
S3. sin frote pericárdico, edema +1 en extremidades bajas. Estudios de laboratorios revelan:
Hb: 8,8g/dL Cuenta leucocitaria; 8,000/mm3 Plaquetas: 186,000/mm3 Na+: 139 mEq/L CI-: 100
mEq/L K+: 7,9 mEq/L HCO3-: 14 mEq/L Nitrógeno de urea: 156 mEq/L Creatinina: 7,2 mg/dL
Glucosa 112mg/dL EKG muestra un ritmo sinusal con elevación de la onda T y ensanchamiento
del complejo QRS, el paciente recibe una ampolla de gluconato de calcio, una ampolla de 50%
de dextrosa con 10 unidades de insulina regular, y una ampolla de bicarbonato de sodio. ¿Cuál
de los siguientes es el próximo paso más apropiado en el manejo del potasio
anormal en el paciente?
a) Administrar Kayexalate (polistireno de sodio)
b) Hemodiálisis de emergencia
c) Administrar 120 mg de furosemida IV
d) Enviar al paciente a casa y organizar transporte para paciente externo de diálisis
e) Detener lisinopril y monitorear niveles de potasio

Respuesta correcta:B

50. Una madre, de 33 años, lleva al médico a su recién nacido de dos semanas para que le
haga una revisión. Nació a término y dio a luz en casa porque sus padres querían un parto
natural. La madre no recibió atención prenatal. No tiene antecedentes de enfermedades graves
y no toma medicamentos. Según la madre, el parto fue rápido y sin complicaciones. Está
siendo alimentado exclusivamente con leche materna. Parece estar sano. Está en el percentil
35 de longitud y en el percentil 40 de peso. La exploración física no muestra anomalías. ¿Cuál
de las siguientes complicaciones tiene mayor riesgo de padecer el recién nacido en este
momento?

A) Hiperparatiroidismo secundario
B) Hemorragia
C) Gastroenteritis
D) Anemia por deficiencia de hierro
E) Leucemia linfoblástica aguda
F) Escorbuto
G) Intusucepción

Respuesta correcta: B (Hemorragia)


51. Un hombre de 24 años es hospitalizado para quimioterapia de inducción, previo a un
trasplante de médula ósea, él tiene una historia de 5 meses de leucemia mieloblástica aguda y
recibe altas dosis de busulfán y ciclofosfamida. Un día después, desarrolló náuseas y vómitos y
no tolera alimentos, ni bebidas ni medicamentos. Su temperatura es de 37.1 °C. ¿Cuál de las
siguientes opciones es el tratamiento farmacológico más apropiado para este paciente?
a) Dopamina
b) Eritromicina
c) Ondansetrón
d) Metoclopramida
e) Omeprazol

Respuesta correcta: C

52. Una mujer de 72 años acude al médico con dolor en la mitad de la espalda desde
hace 3 días. Niega trauma en el área. El examen físico muestra sensibilidad a la
palpación del punto correspondiente a la vértebra T8. Los laboratorios obtenidos son:
Hemoglobina en 13g/dl. Sodio: 137 mEq/L Cloro: 101 mEq/L Potasio: 4.3mEq/L Bicarbonato: 25
mEq/L Creatinina: 0.7 mg/dl Nitrógeno de urea; 21 mg/dl Albúmina=3.6 g/dl Calcio: 9.0 mg/dl
Fosfatasa Alcalina: 50 U/L. Una radiografía de la columna muestra una fractura por compresión
del cuerpo vertebral de T8. La Absorsiometría de Energía Dual de Rayos X (DEXA) muestra un
T-score de densidad mineral ósea de -2.7. ¿Cuál de los siguientes es el diagnóstico más
probable?
a) Osteítis deformante
b) Raquitismo
c) Osteomalacia
d) Osteopenia
e) Osteoporosis

Respuesta correcta: E

53. Un hombre de 65 años llega al departamento de emergencias por una historia de 3 meses
de falta aire progresiva. Niega dolor en el pecho y fiebre. Hace 5 años tuvo mieloma múltiple,
para el cual fue tratado. No toma medicamentos y no tiene alergias. Bebe una cerveza en la
cena diariamente. Su temperatura es de 37.2°C, presión arterial 110/70, pulso de 105 cpm y
frecuencia respiratoria de 22 cpm. Hay distensión venosa yugular hasta el ángulo de la
mandíbula. Los pulmones están claros a la auscultación. El examen cardiaco revela sonidos
cardiacos distantes. Se ausculta S3 y S4. Hay hepatomegalia y edema pedio 2+, Entre sus
laboratorios se observan: Hierro sérico en 100 ug/dL; Capacidad total de unión al hierro en 350
ug/dl y la Ferritina sérica en 100 ng/mL. El EKG muestra disminución de la amplitud de los
complejos QRS. Una radiografía simple de tórax no muestra anormalidades. El ecocardiograma
muestra un miocardio de apariencia moteada. ¿Cuál de los siguientes es el diagnóstico más
probable?

a) Cardiomiopatía alcohólica
b) Amiloidosis
c) Hemocromatosis
d) Cardiomiopatía isquémica
e) Sarcoidosis

Respuesta correcta:B

54. Una mujer de 38 años acude al médico por una erupción en sus piernas desde hace 1
semana. Niega fiebre o prurito. Su historial médico es irrelevante y no toma medicamentos. Su
temperatura corporal es de 36.9°C, presión arterial de 125/85 y pulso de 75 cpm. La saturación
de oxígeno se encuentra en 98% a aire ambiente. El examen físico muestra pápulas
eritematosas elevadas, sensibles a la palpación, sobre sus caras anteriores de las piernas. El
bazo es palpable 3c, por debajo del margen costal izquierdo. La radiografía de tórax simple
muestra infiltrados biliares bilaterales. Un espécimen de biopsia de nódulo linfático hiliar
derecho, muestra granulomas no caseosos. ¿Cuál de los siguientes hallazgos se asocia con
mejor pronóstico para este paciente?
a) Eritema nodoso
b) Bloqueo cardiaco
c) Consolidado pulmonar con nódulos linfáticos biliares
d) Lupus pernio
e) Compromiso ocular

Respuesta correcta: A

55. Un hombre de 63 años llega a urgencias con dolor en su pierna izquierda luego de
resbalarse en el piso de su cocina. El paciente dice haber escuchado un “chasquido” al
impactarse contra el suelo. No ha tenido pérdida de peso ni de apetito. El paciente tiene
historial de diabetes e hipertensión. Su medicación actual consiste en metformina y lisinopril.
Su temperatura es de 36.9°C y su presión arterial es de 130/80. El examen físico muestra
hipoacusia bilateral. Se ausculta un soplo meso-sistólico en crescendo-decrescendo de
intensidad 2/6. El paciente presenta extrema sensibilidad a la palpación del muslo izquierdo, y
moderada sensibilidad a la palpación en el muslo no lastimado. Una radiografía simple del
muslo izquierdo muestra una fractura en el tercio medio de la diáfisis del fémur y un contorno
cortical expandido y deformado. Una radiografía del muslo derecho muestra encorvamiento del
fémur. Un centelleo óseo muestra áreas focales de intensa captación del medio de contraste en
ambos muslos. Laboratorios: Na+: 140 mEq/L K+: 4.8 mEq/L CI-: 105 mEq/L HCO3-; 24 mEq/L
Creatinina: 1.0 mg/dl Fosfato: 2.9 mg/dl AST: 32 Ul/L ALT: 25 Ul/L GGT: 15 Ul/L VES: 18mm/hr
Nitrógeno de urea: 13 mg/dl Además de referirlo a cirugía ortopédica, ¿Cuál de las siguientes
es la farmacoterapia más apropiada?
a) Alendronato
b) Trasplante autólogo de médula ósea
c) Suplementación con calcio
d) Doxorrubicina
e) Altas dosis de corticosteroides

Respuesta correcta: A

56. Una mujer de 28 años G1P0 con 15 semanas de embarazo, viene para su consulta
prenatal de rutina en noviembre. Ella no ha tenido náuseas ni vómitos y a la edad de 15 años,
fue operada por una fractura expuesta, el resto de la historia clínica es irrelevante. Ha recibido
todas las vacunas de la infancia, que se le han ofrecido y su última vacunación fue a la edad de
16 años. Toma habitualmente acetaminofén para cefaleas esporádicas, no fuma, no consume
alcohol ni drogas ilícitas. ¿Cuál de las siguientes opciones, es la vacuna más apropiada que se
le puede administrar a la paciente en esta

a) Vacunas de la Hepatitis A y hepatitis B


b) Vacuna de la hepatitis B y Toxoide tetánico Td
c) Vacuna de la influenza inactivada, MMR y varicela
d) Vacuna de la influenza inactivada y vacuna del neumococo polisacárida
e) Vacuna de la influenza inactivada y toxoide tetánico Td

Respuesta correcta:E

57. Un niño de 9 años, es traído al cuarto de urgencias por su madre porque él dijo que «su
corazón va demasiado rápido y siente que le pesa» Esto le inició hace 3 horas y nunca le había
sucedido esto, de hecho, siempre ha sido un niño saludable, a excepción de enfermedades
comunes de la infancia y no ha visto al doctor en los últimos 2 años. Al auscultarlo, se le
encuentra una Frecuencia cardiaca de 200 x min. Se le convierte fácilmente a ritmo sinusal
después de que su cara es sumergida en solución salina helada por unos cuantos segundos.
En este momento, al examen se le encuentra un soplo holosistólico de grado 3, sobre el lado
izquierdo, junto con un ritmo de galope que se le escucha en el borde esternal izquierdo. Se le
realiza un ECG y tiene frecuencia cardíaca normal, pero muestra un intervalo PR corto con una
elevación lenta del QRS. ¿Cuál de las siguientes opciones, es el diagnóstico más probable?
a) Defecto septal atrial
b) Anormalidad de Ebstein
c) Estenosis pulmonar
d) Tetralogía de Fallot
e) Atresia tricuspídea
Respuesta correcta: B

58. Un hombre de 49 años es evaluado en el cuarto de urgencia por una caída reciente y un
trauma en la cabeza. La tomografía computada es normal y sus lesiones son solamente una
pequeña laceración arriba del ojo derecho. Su historia médica es significante por una gastritis
crónica y osteoartritis, por la cual esporádicamente toma acetaminofén. Sus laboratorios
muestran: Glóbulos rojos 4.1 x 106/ml Hcto; 39% Hb; 12 g/Dl Glóbulos blancos: 7.500/ml VCM:
110/mm 3 Plaquetas: 140,000/mm3 Frotis de sangre periférica: muestra neutrófilos
hipersegmentados Anticuerpos Anti -IF: negativo Fosfatase Alcalina: 50 U/L LDH: 15 U/L AST:
150 U/L Bilirrubina Total: 1 mg/dL ¿Los hallazgos son consistentes con cuál de los siguientes
diagnósticos?
a) Abuso crónico de alcohol
b) Gastritis inducida por drogas
c) Anemia Perniciosa
d) Enfermedad úlcero péptica
e) Hepatitis viral

Respuesta correcta:A

59. Masculino de 32 años sin historia de enfermedad mental tiene "problemas ocupacionales".
Él siente que no avanza profesionalmente en su trabajo como un analista financiero, sin
embargo, él se desempeña por encima de las expectativas en el trabajo que realiza en privado.
Refiere que "se paraliza del miedo" cuando da una presentación en su trabajo. También no
puede concentrarse y desarrolla palpitaciones y temblores cuando se encuentra alrededor de
extraños en las fiestas de su trabajo. ¿Cuál de los siguientes tratamientos es el más apropiado
para prevenir los síntomas y mejorar su desempeño?
a) Benzodiacepinas
b) Buspirona
c) Clomipramina
d) Psicoterapia grupal con sobrevivientes
e) Propanolol

Respuesta correcta:E

60. Un hombre de 32 años acude al médico por dolor de espalda de 4 días de evolución, que
apareció súbitamente durante un estudio diagnóstico. El paciente dice que el dolor se
encuentra “cerca de la columna” al lado derecho. El paciente recientemente comenzó a
levantar peso. Niega fiebre, rigidez matinal, adormecimiento u hormigueo en las extremidades,
debilidad o incontinencia. Refiere beber vino ocasionalmente y no utiliza drogas ilícitas. Su
temperatura es de 36.8°C y pulso de 72 cpm. Al examen físico no se evidencian deformidades
en la columna. El rango de movimiento está limitado por el dolor. No hay puntos de sensibilidad
a la palpación en el área afectada. La fuerza muscular es 4/5 bilateralmente en extremidades
superiores e inferiores, reflejos osteotendinosos normales. No hay sensibilidad a la palpación
en los músculos de la pierna. ¿Cuál de los siguientes es el diagnóstico más probable?
a) Espondilitis anquilosante
b) Herniación de disco intervertebral
c) Bursitis interespinosa lumbar
d) Espasmo de músculo lumbosacral
e) Lesión de articulación sacroilíaca

Respuesta correcta:D

61. Un hombre de 65 años con historial de hipertensión y osteoartritis acude al médico por
debilidad progresiva de 3 años de evolución. Tiene dificultad para levantarse de una silla,
calambres moderados en sus muslos y dificultad para tragar. Niega enfermedad reciente. La
fuerza muscular es de 4/5 bilateralmente en la musculatura proximal de miembros inferiores.
Los reflejos patelares están disminuidos bilateralmente. Laboratorios: Hb: 12.9g/dl
Hematocrito; 44% Leucos: 8,300/mm3 Plaquetas: 255, 000/mm3 Proteína C-Reactiva: 1.2 mg/
dl Actividad de creatina kinasa: 300 U/L ¿Los resultados de cuál de las siguientes pruebas
diagnósticas es más probable que pueda proveer el diagnóstico definitivo en este paciente?
a) Anticuerpos anti-nucleares (ANA)
b) Electromiografía (EMG)
c) Pruebas de función hepática
d) RMN de la columna vertebral y ambas extremidades inferiores
e) Biopsia de músculo

Respuesta correcta:E

62. Un joven de 15 años es llevado al médico por historia de hormigueo episódico en sus
manos, seguidos de decoloración azul y luego roja; desde hace 3 meses. La totalidad del
episodio dura aproximadamente 5 minutos y usualmente ocurre cuando tiene frío. Tiene
cefaleas ocasionales. Niega fiebre, pérdida de peso, debilidad, erupciones, artritis y falta de
aire. Tiene historia de asma inducida por ejercicio, tratada con albuterol. Su padre tiene historia
de hipertensión. El joven juega rugby para el equipo de la escuela. Niega fumar, beber alcohol
y usar sustancias ilícitas. Su temperatura es de 37.2°C, presión arterial de 110/70, pulso de 70
cpm y frecuencia respiratoria de 12 cpm. Su saturación de oxígeno es de 99% a aire ambiente.
El examen físico es normal ¿Cuál de las siguientes opciones es el siguiente paso más
apropiado en el manejo?
a) Solicitar niveles de anticuerpos anti-nucleares (ANA)
b) Solicitar niveles de complemento C3 y C4
c) Microscopía del lecho ungueal
d) Observación
e) Solicitar niveles de Factor Reumatoideo

Respuesta correcta:C

63. Una mujer de 60 años con historia de diabetes mellitus tipo 2 desde hace 30 años, acude al
médico por un pie hinchado. Refiere haber tenido hinchazón, eritema y dolor moderado en su
pie derecho desde hace 3 meses. Ha tenido dificultad para caminar debido a los síntomas.
Tiene historia de hipertensión desde hace 12 años. Sus medicamentos actuales son
metformina, insulina, lisinopril e hidroclorotiazida. Niega fumar, consumo de alcohol y de drogas
ilícitas. Su temperatura es de 36.8°C, pulso de 86 cpm y saturación de oxígeno de 94% a aire
ambiente. Se auscultan crépitos bilaterales bibasales y un S4. Se auscultan soplos carotídeos
bilaterales. El pie derecho está edematoso, indurado y caliente al tacto. Hay un dolor moderado
a la manipulación del pie y disminución de la sensación de pinchazo y estímulo vibratorio
bilateralmente debajo del nivel de la rodilla. HbA1C de 9.1%, Glóbulos blancos en 5,500/mm3.
¿Cuál de los siguientes es el diagnóstico más probable?
a) Enfermedad por depósito de pirofosfato de calcio
b) Celulitis
c) Articulación de Charcot
d) Gota
e) Artritis séptica

Respuesta correcta: C

64. Una mujer de 32 años con hipertensión, acude al consultorio por dificultad para tragar, la
paciente dice que ha tenido disfagia para líquidos y sólidos desde hace 4 meses y ha perdido
aproximadamente 10 libras en ese mismo tiempo- Ella es saludable, a excepción de estos
episodios de disfagia, que se alivian al tomar agua. Al examen, está moderadamente obesa
pero no está intranquila. Su temperatura es de 37'’C, Presión arterial de 137/84 y pulso en 72 x
minuto El resto del examen físico está normal. Se le ordena un estudio de bario que muestra un
encintamiento liso, con forma de punta en el esófago distal y dilatación del esófago proximal.
Se le ordena una endoscopia superior. ¿Cuál de las siguientes opciones, es la explicación más
probable de los síntomas?
a) Acalasia
b) Espasmo esofágico difuso
c) Cáncer esofágico
d) Anillo de Schatzki
e) Esclerodermia

Respuesta correcta:A

65. Una mujer caucásica de 36 años con historia de hipertensión y diabetes mellitus
controlada con dieta, llega al cuarto de urgencias con dolor abdominal de dos días de
evolución. El dolor se ubica en el epigastrio y se irradia a la espalda, tiene algo de náuseas
pero no vómitos y niega diarrea. En los últimos dos años, ha tenido 3 episodios de dolor sordo
en el cuadrante superior que se exacerba al consumir comida grasosa y esto se resolvía
espontáneamente. Toma lisinopril para la presión y niega episodios anteriores de dolor
abdominal y no fuma. Se toma dos cervezas cada fin de semana y la historia familiar no
contribuye. Al examen físico, se ve a una paciente joven con intranquilidad moderada,
temperatura en 37.2°C, Presión arterial en 110/70 mmHg, pulso en 110 X min y respiratorio en
16 x min. Los pulmones están claros a la auscultación y el examen cardiovascular no muestra
soplos, galope ni frote. Al examen abdominal, hay sensibilidad a la palpación en la región
epigástrica sin defensa ni rebote o rigidez. Los laboratorios muestran lo siguiente:
Hemoglobina: 12.6 g/dl Glóbulos blancos: 13,000 x mm3 con 18% de bandas Plaquetas:
370,000 x mm3 Amilasa: 1900 U/l Lipasa: 1600 U/l Calcio: 8.2 mg/dl Triglicéridos: 800 mg/dL
(N: 35-160 mg/dl) AST;80U/L ALT:150U/L ¿Cuál de las siguientes opciones, es el diagnóstico
más probable?
a) Colecistitis aguda
b) Pancreatitis alcohólica
c) Pancreatitis inducida por fármacos
d) Pancreatitis biliar
e) Pancreatitis por hipertrigliceridemia

Respuesta correcta: D

66. Un hombre de 52 años, con hepatitis C y cirrosis, es traído al cuarto de urgencias por su
esposa porque tiene 3 dias de tener dolor abdominal y un día de confusión. El paciente se le
inició furosemida hace una semana por tener edema en las piernas y abdominal. Los otros
medicamentos, incluyen propranolol para las várices esofágicas. Aparenta estar somnoliento
pero responde al llamado, la temperatura está en 37°C, PA en 110/60 mmHg, pulso en 100 por
min y respiratorio en 20 x min. Al examen neurológico, tiene asterixis; tiene el abdomen
distendido con sensibilidad a la palpación difusa, no tiene defensa ni rebote. Tiene matidez
cambiante y edema de tobillo 1+, tiene los niveles de amonio en plasma elevados. Se le
hospitaliza y se le administra lactulosa. ¿Cuál de las siguientes opciones, es el siguiente paso
más apropiado para el manejo?
a) Tomografía de cabeza
b) Ecocardiograma
c) Lorazepam para la confusión del paciente
d) Espironolactona
e) Paracentesis guiada por ultrasonido

Respuesta correcta:E

67. Un hombre negro de 46 años con una historia de trastorno esquizoafectivo es hospitalizado
por psicosis aguda. Presenta alucinaciones auditivas, paranoia, depresión con ideación suicida,
anhedonia, afecto aplanado, apetito aumentado y una ganancia de peso de 10 libras en el
último mes y aumento en la somnolencia. El paciente es estabilizado con haloperidol ¿Cuál de
las siguientes opciones es el mejor antipsicótico atípico para recetar al paciente?
a) Clozapina
b) Olanzapina
c) Quetiapina
d) Risperidona
e) Ziprasidona

Respuesta correcta:E

68. Un hombre de 34 años viene al médico porque tiene dificultad para tragar, progresiva,
desde hace 4 meses. Él ha tenido sensación de que se atora después de comer y también
episodios frecuentes de quemazón en el pecho. En los últimos 10 años, ha presentado
dificultad para tragar alimentos blandos y tiene historia de enfermedad por reflujo
gastroesofágico desde hace 5 años. Su medicación habitual incluye ranitidina, hidróxido de
magnesio y omeprazol. Fumó 1 paquete de cigarrillos diarios durante 15 años y no consume
alcohol. Al examen físico no se muestran anormalidades. ¿Cuál de las siguientes opciones, es
la explicación más probable de los síntomas del paciente?
a) Espasmo esofágico difuso
b) Carcinoma escamoso esofágico
c) Membrana esofágica inferior
d) Constricción péptico-esofágica
e) Esclerodermia

Respuesta correcta:D

69. Un hombre de 40 años acude al médico para la evaluación de cefaleas episódicas desde
hace 5 meses. Los dolores de cabeza afectan a ambas sienes y tienen una intensidad de 4/10.
El paciente ha estado tomando paracetamol, pero los dolores de cabeza no han remitido.
También ha tenido alteraciones visuales, incluyendo visión doble. No tiene náuseas,
intolerancia a la temperatura ni cambios de peso. El paciente no fuma. Bebe 2-3 cervezas los
fines de semana. Está pálido. Su temperatura es de 37°C, su pulso es de 75/min y su presión
arterial de 125/80 mm Hg. El examen oftalmológico muestra una visión periférica deteriorada
bilateralmente. Una resonancia magnética de la cabeza con contraste muestra una masa
intraselar de 16 × 11 × 9 mm. ¿Cuál de los siguientes hallazgos es más probable que aparezca
en la evaluación posterior?

A) Galactorrea
B) Características faciales alteradas
C) Bocio difuso
D) Disfunción erectil
E) Estrías abdominales

Respuesta correcta: D (Disfunción eréctil)

70. Un chico de 16 años acude al médico porque sus padres están preocupados por su
persistente bajo rendimiento escolar. Ha aumentado su impulsividad y tiene dificultades para
hacer y mantener amigos. Al preguntarle, informa de que sus compañeros le acosan por su voz
aguda, su delgadez y sus malas notas. Está en el percentil 94 de altura y en el percentil 50 de
peso. La exploración física muestra ginecomastia bilateral, vello púbico escaso y extremidades
largas en comparación con el tronco. El examen genital muestra testículos pequeños y firmes.
Los estudios séricos muestran un aumento de los niveles de la hormona luteinizante y de la
hormona folículo-estimulante y una ligera disminución de la testosterona sérica. ¿Cuál de los
siguientes es el mayor riesgo de este paciente?

A. Cáncer de próstata
B. Cáncer de mama
C. Disección aórtica
D. Glioma óptico
E. Leucemias agudas

Respuesta correcta: B (Cáncer de mama)

71. Una chica de 16 años es llevada al médico por su madre porque aún no ha tenido la
menstruación. Al nacer, se le encontró una fusión labial parcial y clitoromegalia. Su madre
informa de que durante el embarazo había notado un crecimiento anormal del vello en la
barbilla. La niña tiene un acné severo. Hace tres años se rompió la muñeca tras un pequeño
traumatismo. El año pasado sufrió una fractura por compresión vertebral tras levantar una caja
durante una mudanza. Actualmente toma Isotretinoína oral y un anticonceptivo oral. La
paciente se encuentra en el percentil 97 de altura y en el percentil 50 de peso. Sus signos
vitales están dentro de los límites normales. La exploración física muestra numerosas pústulas
inflamadas en la cara y en la parte superior de la espalda. El desarrollo de las mamas está en
el estadio I de Tanner. La paciente se niega a someterse a un examen pélvico. La ecografía
pélvica muestra múltiples quistes ováricos y un útero normal. ¿Cuál de los siguientes es el
diagnóstico más probable?

A. Síndrome de ovario poliquístico


B. Hiperplasia Adrenal congénita
C. Agenesia Mülleriana
D. Hiperprolactinemia
E. Himen imperforado
F. Deficiencia de Aromatasa
G. Síndrome de Turner

Respuesta correcta: F

72. Un hombre de 58 años es llevado al servicio de urgencias tras sufrir una convulsión tónico-
clónica presenciada. Su mujer dice que ha tenido una tos seca persistente durante 6 meses.
Durante este periodo, también ha tenido fatiga y una pérdida de peso de 4,5 kg (10 libras). El
paciente no tiene antecedentes de enfermedades graves y no toma ningún medicamento. Ha
fumado 1 paquete de cigarrillos diario durante 35 años. Está confuso y se orienta sólo hacia la
persona. Los estudios de laboratorio muestran una concentración sérica de sodio de 119 mEq/L
y una concentración de glucosa de 102 mg/dL. Una radiografía de tórax muestra una masa
irregular y mal delimitada en el hilio derecho. El examen microscópico de esta masa es el que
más probablemente confirmará cuál de los siguientes diagnósticos?

A) Carcinoma de células escamosas de pulmón


B) Carcinoma de células pequeñas de pulmón
C) Carcinoma de células grandes de pulmón
D) Carcinoma bronquial
E) Adenocarcinoma de pulmón

Respuesta correcta: B (Carcinoma de células pequeñas )

73. Una mujer de 29 años acude al médico para que le evalúe la pérdida progresiva de visión
en el ojo izquierdo y la visión borrosa gradual en el ojo derecho durante los últimos 2 meses.
También tiene dolores de cabeza ocasionales y últimamente ha notado una disminución de su
sentido del olfato. Nunca antes había tenido estos síntomas. La única medicación de la
paciente es fexofenadina para las alergias estacionales. Mide 158 cm y pesa 61 kg; su IMC es
de 24 kg/m2. Las constantes vitales están dentro de los límites normales. La agudeza visual es
de 20/40 en el ojo derecho y la percepción de la luz es mínima en el ojo izquierdo. En la prueba
de la linterna oscilante, ambos ojos se dilatan cuando la luz se desplaza del ojo derecho al
izquierdo. La funduscopia muestra un papiledema en el disco óptico derecho y un disco óptico
izquierdo pálido. El resto de la exploración no muestra anomalías. ¿Cuál de los siguientes es el
diagnóstico más probable?

A) Neuropatía óptica hereditaria


B) Pseudotumor cerebral
C) Neuropatía izquémica anterior
D) Meningioma olfatorio
E) Esclerosis múltiple

Respuesta correcta: D (Meningioma olfatorio)

74. Una mujer asiática de 53 años acude al médico porque lleva dos meses con un fuerte dolor
en la pierna derecha al caminar. Solía ser capaz de caminar media milla (800 m) hasta la tienda
de comestibles, pero desde hace un mes es incapaz de caminar 200 metros sin detenerse
debido al dolor. Puede seguir caminando tras una pausa de unos 5 minutos. Tiene hipertensión,
fibrilación auricular y diabetes mellitus de tipo 2. Ha fumado un paquete de cigarrillos diario
durante los últimos 32 años. La medicación actual incluye metformina, enalapril, aspirina y
warfarina. Las constantes vitales están dentro de los límites normales. La exploración muestra
un pulso irregular. La extremidad inferior derecha está más fría que la izquierda. La piel de la
pierna derecha aparece brillante y seca. Los pulsos femorales se palpan bilateralmente; los
pulsos pedios están disminuidos en el lado derecho. ¿Cuál de los siguientes pasos es el más
apropiado para el tratamiento?

A) Resonancia magnética (RMN) de sistema nervioso central (SNC)


B) Ultrasonido Doppler de ambos miembros inferiores
C) Examen de Trendelenburg
D) Examen de conducción nerviosa
E) Índice tobillo brazo.
A) Angiografia

Respuesta correcta: E (Índice tobillo brazo)

75. Una niña de 5 años es llevada al médico por su madre debido a una historia de 3 semanas
de descarga maloliente de la fosa nasal izquierda. También ha tenido un episodio de drenaje de
líquido sanguinolento por la fosa nasal durante este periodo. Lleva 4 días respirando por la
boca mientras duerme. Nació a término. Su hermano de 1 año fue tratado por una
gastroenteritis viral hace 3 semanas. La niña está en el percentil 60 de altura y en el percentil
70 de peso. Su temperatura es de 37,0°C (98,6°F), su pulso es de 96/min, su respiración es de
23/min y su presión arterial es de 96/54 mm Hg. La exploración muestra una secreción
mucopurulenta en la cavidad nasal izquierda. La exploración bucal y otoscópica no es
destacable. La exploración endoscópica de la nariz confirma el diagnóstico. ¿Cuál de los
siguientes pasos es el más apropiado para el tratamiento?

A) Embolizacion tumoral
B) Extracción de cuerpo extraño.
C) Septoplastia
D) Adenoidectomía
E) Tratamiento con glucocorticoides intranasales

Respuesta correcta B (extracción de cuerpo extraño)

76. Una mujer de 46 años acude al médico para un examen rutinario de mantenimiento de la
salud. Se siente bien. Tiene un historial de convulsiones controladas con levetiracetam. Ha
necesitado gafas durante los últimos 13 años. Su padre murió de cáncer de páncreas. Mide
175 cm y pesa 79 kg; su IMC es de 26 kg/m2. Las constantes vitales están dentro de los límites
normales. Se muestra una fotografía de la cara.
Este paciente tiene muy probablemente un mayor riesgo de padecer cuál de las siguientes
enfermedades?

A) Glioma Óptico
B) Cáncer de piel de células escamosas
C) Cáncer gástrico
D) Glaucoma de inicio temprano
E) Enfermedad arterial coronaria
F) Angiomiolipoma renal

Respuesta correcta: E (Enfermedad arterial coronaria)

77. Un niño de 3 meses de edad, previamente sano, es llevado al servicio de urgencias por
sus padres 2 días después de la aparición de fiebre e irritabilidad. Había estado bien hasta
hace 4 días, cuando empezó a alimentarse mal y a dormir más de lo habitual. Su madre dice
que llora cada vez que le cambia el pañal. Sus vacunas están al día. Parece aletargado e
irritable cuando se le despierta para examinarlo. Su temperatura es de 38,4°C (101,4°F), su
pulso es de 110/min y su presión arterial es de 92/58 mm Hg. La exploración física muestra una
pierna izquierda flexionada y rotada externamente; empieza a llorar cuando se le levanta. La
exploración de la cadera izquierda muestra sensibilidad a la palpación y una amplitud de
movimiento limitada; la nalga izquierda está hinchada. El resto de la exploración física no
muestra anomalías. ¿Cuál de los siguientes es el diagnóstico más probable?

A. Necrosis avascular de la cabeza femoral


B. Celulitis
C. Artritis séptica
D. Sinovitis transitoria
E. Displasia de cadera

Respuesta correcta: D (artritis séptica)

78. Un hombre de 37 años acude al médico porque desde hace un mes se le hincha y le duele
la rodilla derecha. No ha sufrido ningún traumatismo en la rodilla ni ha tenido problemas
articulares previos. Tiene hipertensión. Su única medicación es la hidroclorotiazida. Trabaja
como instalador de alfombras. Bebe de dos a tres cervezas al día. Mide 170 cm y pesa 97 kg;
su IMC es de 34 kg/m2. Su temperatura es de 37°C, su pulso es de 88/min y su presión arterial
es de 122/82 mm Hg. La exploración de la rodilla derecha muestra hinchazón y eritema; hay un
edema fluctuante sobre la parte inferior de la rótula. La amplitud de la flexión está limitada por
el dolor. La piel sobre el lugar del dolor no está caliente. Hay sensibilidad a la palpación de la
rótula; no hay sensibilidad en la línea articular. El resto de la exploración no muestra anomalías.
¿Cuál de los siguientes es el diagnóstico más probable?

A) Gota
B) Artritis séptica
C) Bursitis prepatelar
D) Enfermedad de Wilson
E) Osteoartritis

Respuesta correcta: C (bursitis prepatelar)


79. Un hombre de 23 años acude al médico por un temblor en la mano derecha desde hace
tres meses. El temblor ha aumentado en intensidad y no puede realizar sus actividades diarias.
Cuando se despierta por la mañana, su almohada está empapada de saliva. Durante este
periodo, ha sido incapaz de concentrarse en sus clases universitarias. Ha tenido varias caídas
en el último mes. No tiene antecedentes de enfermedades graves. Parece estar sano. Sus
signos vitales están dentro de los límites normales. La exploración muestra una marcha amplia.
Hay un temblor de baja frecuencia que afecta a la mano derecha del paciente en mayor medida
que a la izquierda. Cuando el paciente mantiene los brazos en abducción total con los codos
flexionados, tiene un temblor bilateral de baja frecuencia en los brazos que aumenta en
amplitud cuanto más tiempo mantiene los brazos en alto. La fuerza muscular es normal en
todas las extremidades. La sensibilidad está intacta. Los reflejos tendinosos profundos son 4+
bilateralmente. La dismetría está presente. Se muestra una fotografía del ojo del paciente. El
examen del estado mental muestra un afecto restringido. La velocidad y el ritmo del habla son
normales. ¿Cuál de las siguientes opciones es la farmacoterapia más adecuada?

A) Penicilamina
B) Propanolol
C) Levodopa
D) Deferoxamina
E) Prednisona

Respuesta correcta: A (Penicilamina)

80. Paciente masculino de 66 años con antecedentes de hipertensión arterial y de diabetes


mellitus. Se encuentra en tratamiento por su médico de atención primaria con lisinopril y
metformina con buenos controles actuales. Trabaja habitualmente en un huerto que tiene en su
casa y le gusta practicar deporte. Presenta un IMC de 26kg/m². No consume tabaco ni alcohol.
No consumo de otras drogas ilícitas. El paciente acude a emergencias por astenia y mareo de
tres semanas de evolución junto con sensación presincopal repetida en las últimas horas que
ha hecho que acudiera al hospital junto con su mujer. Decide realizar un electrocardiograma en
que detecta un bloqueo auriculoventricular en segundo grado de Mobitz tipo 1. En su historia
no hay referenias de ninguna afectación en el ritmo cardiaco previamente. ¿Cuál actitud seria
la más indicada?
a) Seguimiento ambulatorio con ECG seriados y estudio Holter de 24 horas antes de tomar
una decisión definitiva
b) Implantación de marcapasos transitorio en la unidad coronaria para posterior
implantación de marcapasos definitivo
c) Ingreso en planta de cardiología e implantación programada de marcapasos
permanente
d) Ingreso en la unidad coronaria y perfusión de atropina hasta que sea posible implantar
marcapasos definitivo
e) Ingreso en la unidad coronaria y colocación de un balón de contra pulsación

Respuesta correcta: C

81. Paciente femenina de 75 años hipertensa y fumadora de 15 cigarrillos al dia que presenta
disnea (estadio funcional I/IV NYHA) de varios años de evolución que se encuentra en
seguimientos por Cardiología en su hospital. Se encuentra en tratamiento con hidroclorotiazida.
No consumo de alcohol. La paciente fue intervenida por un síndrome de túnel del carpo hace
11 años y presento una cesárea. Acude al departamento de emergencias de su hospital por
presentar palpitaciones desde hace 6 días cada vez de mayor intensidad junto con un aumento
de la disnea. En la exploración física encontramos ritmo irregular en la auscultación cardiaca
sin encontrar soplos. El murmullo vesicular se encuentra conservado y presenta roncus
aislados de manera difusa en todos los campos pulmonares. Presenta una presión arterial de
135/100 mmHg. La temperatura corporal axilar es de 36.8°C (98.2 F). Se realiza un ECH que
muestra una taquicardia de QRS estrecho arrítmica a 140 latidos por minuto. La paciente esta
eupneica. ¿Cuál de las siguientes medidas es la más correcta?
a) Cardioversionelectrica sincronizada seguida de anti coagulación durante 4 semanas
b) Anti coagulación durante 3 semanas, cardioversión y anti coagulación indefinida
c) Control de la frecuencia cardiaca con digoxina y cardioversión dentro de 3 semanas con
anti coagulación 4 semanas mas
d) Antiagregacion con AAS durante 3 semanas, cardioversión y antiagregacion durante 4
semanas mas
e) Control de la frecuencia con atropina y antiagregacion con aspirina sin necesidad de
cardioversión
f) Antiagregacion y posterior seguimiento del paciente esperando su reversión
espontanea

Respuesta correcta: B

82. Varón de 67 años que acude a emergencias por presentar desde hace dos semanas
sensación de disnea. El paciente refiere que no había presentado previamente esa
sintomatología. No presenta diagnósticos previos y no toma ningún fármaco. Fumador de20
cigarrillos al día desde los 15 años de edad. Consumo de dos cervezas de alcohol al día. No
consume otras drogas. En la amnesis el paciente refiere disnea de mínimos esfuerzos siendo
incapaz de subir las escaleras de su casa ni de caminar durante mas de 30 segundos. Además
el paciente presenta ortopnea que le obliga a permanecer sentado. Ademas desde hace 4 dias
presenta oliguria. En la exploración física encontramos mal estado general y el paciente esta
pálido tanto a nivel de la piel como de las mucosas, frio y sudoroso. A la auscultación, el
paciente esta taquipneico, y se escuchan crepitantes húmedos en ambos pulmones. Presenta
las siguientes constantes: Presión arterial de 85/55 mmHg, temperatura axilar de 36.5°C (97.7
F). Se decide realizar un ECG en el que encontramos una taquicardia sinusal a 120 lpm. Se
realiza una radiografía de tórax en la que se observa un infiltrado pulmonar bilateral, con
distribución en alas de mariposa. Ante el diagnóstico del paciente, de entre las siguientes
medidas, ¿Cuál NO estaría indicada inicialmente?
a) Oxígeno al 50%
b) Cloruro Morfico
c) Cama a 45°
d) Furosemida
e) Nitroglicerina iv

Respuesta correcta: E

83. Un paciente de 70 años es llevado al departamento de emergencias por sus familiares


debido a 3 horas de intenso dolor torácico irradiado a interescapular que refiere que se le clava
como un cuchillo. El paciente describe el dolor como agudo. El nunca ha experimentado este
tipo de dolor. Sus otros problemas médicos incluyen una historia de 20 años de hipertensión y
una historia de 10 años de diabetes mellitus tipo 2. El no usa tabaco ni alcohol. El paciente
parece ansioso e incomodo. Su presión arterial es de 189/110 mmHg en el brazo derecho y de
181/112 mmHg en el izquierdo, el pulso es de 105 lpm y las respiraciones son de 18 por
minuto. El examen muestra un soplo diastólico decrescendo temprano que se escucha mejor
en el borde esternal izquierdo del cuarto espacio intercostal. El ECG muestra la taquicardia
sinusal, los criterios de voltaje para la hipertrofia ventricular izquierda y las inversiones de la
onda T en las derivaciones V5 y V6. La creatinia serica es de 3.1 mg/dL. ¿Cuál de os
siguientes es el mejor paso siguiente en el manejo de este paciente?
a) Angiografia coronaria
b) Niveles de troponina
c) Angiografia por RM
d) Ecocardiografia transesofagica
e) BNT en suero
f) Angio –TC

Respuesta correcta: D
84. Acude a su consulta una mujer de 68 años, con hipercolesterolemia en tratamiento dietético
con buen control por parte de su médico de atención primaria. Además, la paciente toma de
forma diaria levotiroxina por presentar hipotiroidismo de origen autoinmune. Su madre
presentaba también hipotiroidismo de origen autoinmune. Su madre presentaba también
hipertiroidismo y vitigilo que aparece en una de sus hermanas. Acude en el momento axtual a
su consulta presentando clínica de disnea de esfuerzo desde hace 9 meses. Desde ese
momento refiere que le cuesta realizar las actividades que venía haciendo antes ya que le falta
el aire. Refiere que su disnea ha progresado los últimos meses. No es fumadora, aunque
durante su infancia su padre fumaba mucho en su casa. No consumo de alcohol ni de drogas
ilícitas. Presenta un peso de 63 kg (138.9 lb) y una altura de 161 cm. A la exploración física
presenta un ritmo sinusal con una frecuencia de unos 70 latidos por minuto y además aparece
un soplo diastólico en 2֯ espacio intercostal derecho, así como 3 y 4 tono. El pulso arterial es
bífido. ¿Cuál es su sospecha diagnostica en este caso?
a) Miocarditis
b) Insuficiencia aortica
c) Estenosis mitral
d) Regurgitación de válvula pulmonar
e) Insuficiencia tricuspidea
f) Miocardiopatía hipertrófica apical
g) Insuficiencia mitral

Respuesta correcta: B

85. Un niño afroamericano de 1 año de edad es llevado a la evaluación rutinaria por parte de su
Pediatra de atención primaria. Está aprendiendo a caminar y dice "mamá" y "papá".
Recientemente hizo la transición de la leche materna a la leche de vaca. La revisión de los
sistemas es inicialmente normal y el examen físico muestra un niño bien parecido y bien
alimentado sin anormalidades. Sin embargo, ambos padres tienen un rasgo drepanocítico, su
hermana anemia de célula falciformes y los resultados de laboratorio son los siguientes
Contaje sanguíneo completo Hemoglobina 14 g/dl Hematocrito 42% Volumen corpuscular
medio 88 fL. Reticulocitos 2% Electroforesis: hemoglobina A 60% Hemoglobina S 0%
Hemoglobina 40% Hemoglobina 0% ¿Cuál es la complicación más común que podría tener
este niño?
a) Síndrome coronario agudo
b) Dactilitis.
c) Hematuria.
d) Accidente cerebrovascular isquemico
e) Osteomielitis.
f) Infarto esplénico

Respuesta correcta: C

86. Paciente masculino de 59 años, ¡previamente sano y sin antecedentes -médicos ni


quirúrgicos de interés, ingresa con dolor retrosternal severo, en reposo de 3 horas de Curación
que se irradia a mandíbula en el departamento de emergencias de su hospital. En su llegada
se realiza de manera urgente un ECG que muestra elevación marcada de ST II, III y aVF.
Presenta presión arterial de 145/95 mmHg con frecuencia cardíaca de 105 latidos por minuto.
En la analítica la troponina se encuentra muy elevada. Unas horas después, durar te su
estancia en urgencias el paciente desarrolla oliguria e hipotensión (presión arterial de 85/30
mmHg). Se le coloca un catéter de Swan-Ganz y se miden las siguientes presiones: Presión
capilar pulmonar enclavada: 4 mmHg Presión libre en la arteria pulmonar: 22/4 mmHg Presión
medía de la aurícula derecha: 11 mmHg. ¿Cuál de los siguientes tratamientos es el más
adecuado para este paciente?
a) Líquidos i.v.
b) Digoxina i.v.
c) Noradrenalina i.v.
d) Dopamina i.v.
e) Furosemida i.v.
f) Balón de contrapulsación intraaórtico.

Respuesta correcta:A

87. Un paciente masculino de 62 años es llevado al departamento de emergencia debido a


fiebre, dolor de cabeza, confusión y vómitos durante /os últimos 2 días. Su historial médico es
significativo por trasplante de hígado secundario a enfermedad de Wilson, hipertensión y
diabetes mellitus tipo 2. Los medicamentos del paciente incluyen insulina de acción
prolongada, amlodipina y tacrolimus; él no tiene alergias a los medicamentos. La temperatura
es de 39.2 °C (102.6 F), la presión arterial es de 120/75 mm Hg, el pulso es de 102 latidos por
minuto y las respiraciones son de 18 por minuto. Él está despierto pero confundido. El fondo de
ojo no muestra anormalidades. Su cuello está rígido. Los exámenes cardíacos y pulmonares
son normales. El recuento de glóbulos blancos es de 17,000/mm3 con predominio de neutrófilo
73. La creatinina sérica es de 1.1 mg/dL. Los cultivos de sangre están pendientes. La
tomografía computarizada de la cabeza no tiene nada especial. Los resultados de punción
lumbar están pendientes. ¿Cuál de las siguientes es la terapia antibiótica empírica más
apropiada para este paciente?
a) Cefepima y ampicilina.
b) Cefepima, vancomicina y ampicilina.
c) Ceftazidima y vancomicina.
d) Ceftriaxona.
e) Meropenem y vancomicina.

Respuesta correcta:B

88. Un niño de 15 años de edad con enfermedad de células falciforme.; lleva varios días de
fiebre y dolor severo en la rodilla izquierda. El pecie-te fue hospitalizado previamente con crisis
vasooclusivas, que generalmente ocurren en sus piernas. Su temperatura es de 38.2 °C (100.8
F). El examen físico muestra sensibilidad e hinchazón marcadas sobre la tibia proximal. Los
estudios de laboratorio revelan leucocitosis elevada, así como la proteína C.-reactiva. La
resonancia magnética de las piernas muestra una marcada inflamación de la tibia proximal sin
derrame articular. Se obtiene hemocultivo y se administran antibióticos intravenosos. ¿Cuál, de
los siguientes organismos es la causa más probable de la condición de este paciente?
a) Escherichia coli.
b) kingella kingae.
c) Neisseria gonorrhoeae.
d) Pseudomonas aeruginosa.
e) Salmonella enteritidis
f) Streptococcus agalactiae.
g) Streptococcus pneumoniae.

Respuesta correcta:E

89. Paciente masculino de 64 años, diabético en tratamiento con antidiabéticos orales del tipo
de la sinagliptina además de la metformina. El paciente es valorado en el servicio de urgencias
de su hospital por presentar una angina inestable. Después del tratamiento agudo el paciente
es valorado a nivel de consulta y tras programarlo, se le realiza una coronariografía
visualizando enfermedad de las tres arterias coronarias. Se opta por cirugía de
revascularización míocárdica. ¿Cuál de los siguientes injertos es el más adecuado para
revascularización coronaria por mayor permeabilidad a largo plazo?
a) Arteria mamaria interna izquierda.
b) Vena safena mayor.
c) Arteria radial.
d) Arteria gastroepiploica.
e) Arteria upigástrica inferior.
f) Vena safena menor.
g) Arteria cubital.

Respuesta correcta:A

90. Paciente femenina de 19 años, sin antecedentes de interés que acude a su consulta por
"intolerancia al frío". La paciente refiere que cuando ocurre con temperaturas bajas presenta
cianosis y dolor intenso en las zonas acras sobre todo en los dedos de la mano, aunque
también a nivel de orejas y en la punta de la nariz. Refiere que presenta esta situación desde
hace más de 5 años. La paciente no consume alcohol ni tabaco. Su madre refiere que
presentaba hipotiroidismo pero que falleció hace 3 años por un cáncer de mama con presencia
de metástasis a distancia. Ella se encuentra muy preocupada por la posibilidad de que puede
ser la manifestación de un tumor. Refiere que la autoexploración mamaria no ha notado
ninguna tumoración. En la exploración cervical no se palpa aumento de tamaño de la glándula
tiroides ni la presencia de tumoraciones en la misma. Niega el consumo de drogas ilícitas. Ante
la clínica de la paciente, teniendo en cuenta su sospecha diagnóstica, señale cuál de las
siguientes opciones se asocia a la enfermedad en cuestión:
a) Esclerodermia.
b) Psoriasis.
c) Uso de beta agonistas.
d) Uso de calcio antagonistas.
e) Hiperhidrosis.
f) Cáncer de mama.

Respuesta correcta:A

91. Una paciente femenina de 36 años llega a la consulta debido a pérdida de peso. Ella ha
perdido 5 kg (11 lb) en los últimos 3 meses asociados con palpitaciones, intolerancia, temblores
persistentes y sudoración excesiva. Los historiales médicos y familiares pasados no tienen
nada especial y el paciente no usa tabaco, alcohol o drogas ilícitas. Ella está casada y tiene un
hijo. La presión arterial es 140/70 mm Hg y el pulso es 104 latidos por minuto y regular. El
examen físico muestra un nódulo de 2x2 cm en el lóbulo tiroideo izquierdo. El resto de la
glándula tiroides es normal y no presenta una linfadenopatía cervical asociada. El examen de
los ojos muestra un retraso mínimo del párpado, pero no proptosis. Hay un temblor fino en
ambas extremidades superiores. Los reanálísis de la prueba de la función tiroidea son los
siguientes: TSH 0.03 pu/m T3 14 ug/dL T4 330 ng/dl La gammagrafía con yodo radiactivo
muestra la captación solo en el nódulo. La paciente desea tener hijos adicionales y le preocupa
que no pueda quedar embarazada si recibe tratamiento por su estado actual. ¿Si no se trata,
este paciente corre el mayor riesgo de desarrollar cuál de las siguientes complicaciones?
a) Pérdida ósea.
b) Enfermedad de la arteria coronaria.
c) Hipertiroidismo fetal.
d) Cáncer de tiroides.
e) Pérdida de la visión.

Respuesta correcta:A

92. Paciente masculino de 59 años con antecedentes de hipertensión arterial desde hace más
de diez años y fumador habitual de 30 cigarrillos al día. Su tratamiento incluye lisinopril e
hidroclorotiazida. Bebedor de tres copas de vino al día. No consumo de otras drogas. Su padre
falleció súbitamente a los 62 años sin que fuera posible conocer la causa de la defunción.
Acude a su hospital por dolor torácico muy intenso de inicio brusco 3 horas antes que se ha
desplazado a la región interescapular. Realiza una exploración en que encuentra palidez,
sudoración profusa junto con presión arterial de 180/108 mmHg, frecuencia cardíaca de 108
latidos por minuto y pulsos radiales muy disminuidos. Encuentra en la auscultación un soplo
liastólico en borde esternal izquierdo. El ECG revela taquicardia sinusal con aumento del QRS
en varias derivaciones y descenso del ST y T negativa asimétrica en 1, aVL y de V4 a V6.
¿Cuál es el diagnóstico más probable?
a) Síndrome coronario agudo.
b) Miopericarditis aguda.
c) Tromboembolismo pulmonar.
d) Edema agudo de pulmón.
e) Disección aórtica aguda.
f) Neumctórax espontáneo.

Respuesta correcta:E

93. Un paciente masculino de 43 años acude a la consulta debido a un historial de 6 meses de


dolor intermitente en el abdomen superior asociado con náuseas. El paciente describe
episodios de dolor epigástrico opaco que generalmente empeoran 15-30 minutos después de
las comidas y duran unas pocas horas. El dolor no se alivia con antiácidos, pero mejora al
inclinarse hacia adelante. Él también ha tenido ocasionalmente diarrea con heces amarillentas.
El paciente ha perdido 6.8 kg (15 lb) en los últimos 12 meses. Hace cinco años, fue
hospitalizado por 3 días con dolor abdominal agudo. Él no tiene otros problemas médicos y no
toma medicamentos. El paciente fuma un paquete de cigarrillos al día y consume alcohol casi a
diario. Su historia familiar es significativa para la diabetes mellitus en su madre y el cáncer de
próstata en su padre. ¿Cuál de los siguientes indicará probablemente un diagnóstico para la
condición de este paciente?
a) Anticuerpos contra la transglutaminasa tisular.
b) Antígeno asociado al cáncer 19-9 niveles.
c) Tomografía computarizada del abdomen.
d) Angiograma mesentérico.
e) Niveles de lipasa sérica.
f) Endoscopia gastrointestinal superior.

Respuesta correcta:C

94. Paciente femenina de 56 años con historia de eccema seborreico y psoriasis desde
infancia. Sus tratamientos incluyen clobetasol y calcipotriol en espuma y en pomada. Además,
la paciente en los últimos 5 años presenta cuadro compatible con liquen plano pilar en cuero
cabelludo que comenzó como comedón y papulas rojizas en zona de vértex. Acude ahora a
servicio de urgencias por reactivación de herpes labial en días previos que ahora se acompaña
de lesiones dianiformes en dorso de manos y se llega al diagnóstico de eritema exudativo
multiforme. Evaluamos a la paciente y presenta placas de alopecia lisas sin folículo
diseminadas por cuero cabelludo. La paciente se encuentra muy preocupada porque a pesar
de haber seguido las indicaciones de su dermatólogo en el tratamiento de todas sus
enfermedades cutáneas estas lesiones en el cuero cabelludo han evolucionado y no
desaparecen dando lugar a una alopecia cicatricial definitiva ¿Cuál de los siguientes procesos
puede ser la causa de esta situación?
a) Psoriasis.
b) Liquen.
c) Eritema exudativo multiforme minor
d) Eczema seborréico de cuero cabelludo.
e) Alopecia androgénica.
f) Efluvio telogenico

Respuesta correcta:B

95. Un niño de 8 años previamente sano acude a consulta por un cuadro de febrícula de 4 días
de evolución, con dolor a la deglución. El niño no había presentado previamente enfermedades
de interés a excepción de un cuadro de amigdalitis con el desarrollo de un absceso
periamigdalino que requirió drenaje quirúrgico en urgencias de su hospital y tratamiento
antibiótico. No consume ningún fármaco de forma habitual. Se encuentra correctamente
vacunado para su edad. Se encuentra en percentil 30 de peso y percentil 40 de talla para su
edad. El niño refiere que el dolor es grande y que aumenta en función el tamaño del alimento
que se quiere ingerir. Disminuye cuando se ingieren líquidos fríos. En la exploración física
encontramos lesiones erosivas en el paladar. Además, exploramos de forma completa al niño y
encontramos vesículas intraepidérmicas que afectan a las palmas y plantas. Entre los
siguientes diagnósticos ¿cuál es el más probable?
a) Eritema multiforme.
b) Rickettsiosis.
c) Síndrome de Steven-Jonhson.
d) Enfermedad de pie, mano, boca.
e) Varicela.
f) Gripe.

Respuesta correcta: D

96. Una paciente femenina de 14 años presenta desde hace 5 días una erupción cutánea
generalizada muy pruriginosa. Acude a su consulta refiriendo que el prurito que presenta es de
gran molestia y que le impide realizar su vida normal. La paciente no presenta antecedentes de
interés dermatológicos y es intolerante a la leche de vaca por lo que consume leche de soja de
forma habitual desde la infancia. Además, refiere que presenta alergia de contacto al níquel. No
consume ningún fármaco de forma habitual ni ninguna droga ilícita. No consumo de alcohol ni
tabaco. Está correctamente vacunada y siempre ha vivido en San José junto con sus padres y
dos perros. Exploramos a la paciente y la lesión está formada por placas eritemato-edematosas
de entre 2 y 14 centímetros de diámetro que no presentan descamación y tendencia a adquirir
una morfología anular. Las lesiones desaparecen en menos de 24 horas apareciendo otras
nuevas. Las mucosas están respetadas. ¿Cuál será su sospecha diagnóstica?
a) Ricketsia.
b) Rubeola.
c) Toxicodermia.
d) Shock tóxico estafilocócico.
e) Varicela.
f) Urticaria.
g) Secundarismo sifilítico.
Respuesta correcta:F

97. Una paciente de 36 años, acude a la consulta con frecuentes dolores de cabeza y
amenorrea durante los últimos 2 meses. También ha tenido pérdida de peso, palpitaciones
intermitentes y vagas alteraciones visuales. Su historial médico no tiene nada especial y no
toma medicamentos. La paciente experimentó la menarquía a los 12 años y ha tenido ciclos
menstruales normales durante toda su vida. Ella no fuma cigarrillos ni bebe alcohol. La presión
arterial es 130/60 mm Hg, y el pulso es 103 latidos por minuto y regular. El peso es 69 kg (152
Ib) y hace un año era de 76 kg (168 Ib) El examen muestra una glándula tiroides no sensible,
agrandada simétricamente. La auscultación del tórax muestra taquicardia con ritmo cardíaco
regular. Se nota un fino temblor de mano. El resto del examen físico no es notable. Los
resultados de laboratorio son los siguientes Suero T3 13.9 ug /dL Suero T4 6 pU/mL Suero TSH
222 ng/dL (0,5-5) ¿Cuál de los siguientes es el diagnóstico más probable para este paciente?
a) Resistencia generalizada a la hormona tiroidea.
b) Enfermedad de Graves.
c) Tiroiditis de Hashimoto.
d) Embarazo normal.
e) Ingesta de hormona tiroidea subrepticia.
f) Adenoma hipofisario secretor de TSH.

Respuesta correcta:F

98. Paciente femenina de 17 años que acude a su médico de atención primaria por pérdida de
peso en los últimos tres meses desde a pesar de un aumento de la ingesta, aumento de la
sudoración y palpitaciones. Ha pasado de un peso de 61 kg (134,4 Ib) a uno de 54 kg (119 Ib).
La paciente mide 166 cm de altura. La paciente no tiene, antecedentes médicos ni quirúrgicos
de interés. Presentó la menarquia a los 13 años de edad y presenta ciclos regulares sin dolores
intensos durante la menstruación. No consume alcohol' ni tabaco. En la exploración física
encontramos una glándula tiroides aumentada de tamaño de forma homogénea no dolorosa a
la palpación. Se realiza una analítica en la que encontramos niveles en suero de:
T4L de 7 pU/ml TSH <0,05 ng/dl Se realiza una gammagrafía tiroidea que capta de forma
difusa. Ante el diagnóstico realizado se decide comienzo de tratamiento médico. A las 48 h la
paciente acude a urgencias de su hospital porque presenta fiebre de 41 °C (105.8 F) y
odinofagia. Señale cuál será su actitud en este caso:
a) Realizar hemograma urgente para ver la fórmula leucocitaria.
b) Aumentar la dosis de antitiroideos.
c) Administrar urgentemente I131.
d) Diagnosticarla de tiroiditis e iniciar tratamiento con AAS.
e) Tiroidectomía urgente.
f) Tratamiento antibiótico intravenoso urgente por sobreinfección bacteriana.

Respuesta correcta:A

99. Paciente masculino de 35 años que acude a su consulta por presentar obesidad de
predominio abdominal junto con estrías rojizas en el abdomen. El paciente refiere que esta
clínica ha ido comenzando en los últimos dos años y que él previamente no presentaba este
acúmulo graso. El paciente refiere que no ha aumentado la cantidad da comida que ingiere. El
paciente pesa 84 kg (185 lb) y mide 178 cm. No presenta antecedentes personales ni familiares
de interés. No consume tabaco. Bebedor de 4-5 cervezas a la semana. No consumo de drogas
ilícitas. Se explora al paciente y presenta obesidad de predominio central con presencia de
acúmulo graso a nivel cervical posterior. Además, presenta estrías rojas-violáceas en abdomen.
En las extremidades inferiores el paciente presenta hematomas de tamaño variable que refiere
que le han aparecido con mínimos traumatismos. Presenta una presión arterial de 155/105
mml-Ig y frecuencia cardíaca de 86 latidos por minuto. Temperatura exilar de 35.7 'C (96.2 F).
Se le realiza una analítica en la que destaca: Na: 149 mEq/I K: 3'2 mEq/I En el hemograma
encontramos poliglobulia, neutrofilía y linfopenia. Presenta cortisolurias elevadas en dos
ocasiones, ausencia de frenado débil con dexametasona y frenado del cortisol superior al 90%
en la prueba de frenado fuerte. La siguiente prueba a realizar en este paciente para llegar al
diagnóstico etiológico es:
a) Catererismo de senos petrosos.
b) RM hipotálamo-hipofisaria.
c) TAC torácico-abdomino-pélvico.
d) Cateterismo de senos petrosos.
e) Detección de niveles de fármacos en sangre.

Respuesta correcta:B

100. Paciente femenina de 52 años que acude a su médico de atención primaria para una
revisión rutinaria. La paciente presenta hipertensión arterial en tratamiento con ibesartán.
Además, la paciente presenta artrosis en ambas rodillas que le obliga a tratamiento analgésico
múltiple habitual. Es obesa presentando un 'MG de 37,2 kg/m2. Actualmente no fumadora, tras
haber abandonado el hábito tabáquico hace 10 años por la defunción de su hermano mayor por
un carcinoma epidermoide de pulmón. No consumo de alcohol ni de drogas ilícitas, En su
exploración presenta una presión arterial de 135/85 mmHg con frecuencia cardíaca de 75
latidos por minuto. Frecuencia respiratoria de 18 respiraciones por minuto. Se realiza un control
analítico para despistaje de diabetes mellitus y se encuentra que presenta en dos
determinaciones analíticas valores de glucemia por encima de 126 mg/dl, por lo que es
diagnosticada de la enfermedad. Tras 4 meses de tratamiento dietético y ejercicio físico regular
continua con cifras elevadas en su glucemia usted se plantea instaurar tratamiento con
antidiabéticos orales. Señale cual es el antidiabético oral recomendado como primera opción
en esta paciente:
a) Inhibidores de la alfa glicosidasa.
b) Insulina.
c) Sulfonilureas.
d) Incretinas.
e) Metlormina.
f) Glitazonas.
g) Atorvastatina.

Respuesta correcta:E

101. Un paciente masculino de 42 años acude al departamento de emergencias debido a dolor


de espalda severo después de una caída. Se tropezó al salir de su automóvil y cayó. El
paciente tiene 20 años de evolución de una espondilitis anquilosante, por lo que toma la
medicación prescrita con regularidad, pero nunca ha tenido dolor similar a este ya que este es
más localizado y de mayor intensidad. Los signos vitales son normales a excepción de una
taquicardia leve y regular. Parece estar dolorido, con muecas y diaforesis. La auscultación
cardíaca muestra un soplo diastólico temprano. Tuvo además úlcera duodenal hace 7 años. El
paciente no usa alcohol, tabaco. A la exploración presenta en la palpaciónde las apófisis
espinosas un dolor en la línea de la región lumbar superior. El rango de movimiento espinal
está restringido. No hay dolor sobre las articulaciones sacroilíacas en este momento ¿Cuál de
las siguientes es la causa más probable de su dolor actual?
a) Hernia discal.
b) Dolor visceral referido.
c) Metástasis espinal.
d) Esponditis inflamatoria.
e) Espondilolistesis.
f) Fractura vertebral.

Respuesta correcta:F

102. Un niño de 10 días de edad, por lo demás sano, es llevado al médico por sus padres
debido a un aumento progresivo de las mamas bilaterales desde hace 4 días. Los padres
informan de que, desde ayer, han notado una secreción de pequeñas cantidades de un líquido
blanco por la mama izquierda. Durante el embarazo, la madre fue diagnosticada de
hipotiroidismo y fue tratada con L-tiroxina. La abuela materna de la paciente murió de cáncer
de mama. La paciente pesa actualmente 3100 g (6,8 lb) y mide 51 cm (20 in). Los signos
vitales están dentro de los límites normales. La exploración muestra unas mamas
simétricamente aumentadas, no sensibles, con pezones invertidos bilateralmente. El resto de la
exploración no muestra anomalías. ¿Cuál de los siguientes pasos es el más apropiado para el
tratamiento de esta paciente?

A) Suplementar con testosterona


B) Tranquilizar y explicar a la madre que esto es normal
C) Biopsia de mama
D) Medición de gonadotropinas
E) Medición de TSH
F) Análisis de cromosomas

Respuesta correcta: B

103. Acude al departamento de emergencias de su hospital una paciente femenina de 20 años


por deterioro del estado general y necesidad de respirar profundamente, que ha aparecido en
las últimas horas. La paciente presenta como antecedente personal la presencia de asma
extrínseco en tratamiento con fluticasona y con salbutamol. Además, la paciente presenta
desde la infancia dermatitis atópica por la que ha necesitado gran cantidad de cremas a lo
largo de su vida, aunque actualmente refiere que se encuentra, muy bien controlada. Refiere
una pérdida de peso en los 3 últimos días de más de 4 kg a pesar de que se encuentra con
gran necesidad de comer durante todo el tiempo. Refiere que presenta poliuria, una gran
sensación de sed y náuseas. No presenta tos ni sensación febril. En la exploración presenta
aspecto de gravedad, presión arterial 110/60 mmHg, respiración profunda y rápida (29
respiraciones por minuto), frecuencia cardíaca de 110 latidos por minuto y temperatura de 36,2
°C (97.1 F). El nivel de conciencia se encuentra conservado y presenta sequedad de mucosas
presentando signo del pliegue positivo. Realizamos una analítica en la que encontramos:
Glicemia 420 mg/dL Na+ 131 mEq/L K+ normal pH 7,08 B i c a r b o n a t o 8 m E q / L
Cetonuria (+++) Señale su sospecha dignóstica:
a) Debut de una diabetes mellitus tipo 2 con disnea probablemente por neumonía o
tromboembolismo,y a que toma anovulatorios.
b) Descompensación por como hiperglucémico hiperosmolar en paciente con debut
diabético.
c) Debut de diabetes tipo 1 con cetoacidosis.
d) Cetoacidosis alcohólica.

Respuesta correcta C
104. Un paciente masculino de 62 años acude por hemorragia digestiva en forma de
hematemesis al departamento de emergencias de su hospital. El paciente presenta
antecedentes de hipertensión arterial y de diabetes mellitus no insulin-dependiente. Se
encuentra en tratamiento habitual con lisinopril y con metformina y glicazida con seguimiento
por parte de su médico de atención primaria con mal control de las dos enfermedades.
Fumador de más de un paquete al día y bebedor de más de 160 g de alcohol al día desde hace
más de 30 años. El paciente se encuentra con dificultades a la anamnesis. Exploramos al
paciente, que no precenta signos de ascitis aunque sí de alcoholismo crónico. Realizamos una
analítica con: Hb 10 gr/dI Leucocitos 8500/mm3 Plaquetas 90.000/mm3 Bilirrubina total 3
mg/dl AST 160 U/L ALT 60 U/L Albúmina 2,2 g/dl índice de Quick 60% Se reanimó al enfermo y
se realiza una endoscopia que objetiva varices esofágicas grado I, múltiples manchas rojizas
pequeñas a veces confluentes en estómago, más prominentes en antro. ¿Cuál tratamiento al
alta es fundamental en este px ente para evitar la recidiva?
a) Omeprazol.
b) Seguimiento endoscópico periódico.
c) Esclerosis endoscópica de las varices.
d) Erradicación antibiótica del Helicobacter Pylorii.
e) Betabloqueantes no cardioselectivos como propranolol.

Respuesta correcta:E

105. Un jubilado de 70 años viene a su consulta porque presenta dificultad para oír. Su esposa
dice que ha estado elevando el volumen de la televisión mucho más fuerte recientemente. El
paciente afirma que puede escuchar bien cuando habla con los miembros de su familia en
casa, pero tiene una dificultad significativa para escuchar en restaurantes o durante otras
reuniones familiares, razón por la cual prefiere quedarse en casa la mayor parte del tiempo.
Trabajó en un astillero durante 30 años y se retiró hace cinco años. Él no tiene un historial de
exposición significativa al ruido. ¿Cuál es el diagnóstico más probable?
a) Otosclerosis.
b) Presbiacusia
c) Efusión del oído medio.
d) Enfermedad de Meniere.
e) Depresión del neuróma acústico.

Respuesta correcta:B

106. Evaluamos en una revista científica que un grupo de investigadores realizó una evaluación
prospectiva acerca de la eficacia de des, alternativas en el tratamiento de las infecciones óticas
de repetición en niños menores de tres años. De forma aleatorizada, 51 niños recibieron
amoxicilina-ácido clavulánico por vía oral y 49 niños no recibieran tratamiento farmacológico,
sólo se realizó en ellos observación. Se evaluó en ellos la disminución de la secreción ática.
¿Qué tipo de estudio epidemiológico realizaron los investigadores?
a) Estudio de cohortes.
b) Estudio de casos y controles.
c) Ensayo clínico antes-después.
d) Ensayo de campo,
e) Ensayo clínico aleatorizado.
Respuesta correcta:E

107. Un niño de 11 años es llevado a la consulta por sus padres debido a una erupción cutánea
pruriginosa durante 2 semanas. La erupción que comenzó en sus manos posteriormente se
extendió al tronco y las extremidades inferiores. Su madre informa que el paciente
constantemente se rasca la erupción y su sueño ahora se ve perturbado debido a un prurito
intenso. Un primo que recientemente visitó y compartió una habitación con él tuvo síntomas
similares. El paciente no ha tenido fiebre y está al día con las vacunas de rutina. El examen
físico muestra numerosas pápulas y vesículas eritematosas de 2 a 3 mm con excoriación en el
cuello, las manos, los pies y la cintura. Su pecho y brazos y piernas proximales no se ven
afectados. ¿Cuál de las siguientes es la intervención inicial más apropiada en el manejo de
este paciente?
a) Aciclovir oral.
b) Clindamicina oral.
c) Clotrimazol tópico.
d) Hidrocortisona tópica
e) Permetrina tópica.

Respuesta correcta: E

108. Una paciente femenina de 27 años adicta a drogas por vía endovenosa tipo heroína y
portadora de VIH debuta con ictericia brusca, náuseas, vómitos y astenia por lo que acude al
departamento de emergencias de su hospital. Tiene antecedentes de hepatitis crónica C
adquirida por vía endovenosa hace diez años y en ocasiones sigue compartiendo jeringuillas.
En la exploración física la paciente presenta coloración amarillenta a nivel de piel y mucosas a
nivel conjuntival. La presión arterial es de 95/55 mmHg con una frecuencia cardíaca de 95
latidos por minuto. La temperatura exilar es de 36,3 °C (97,3 F). Se realiza en el paciente una
analítica que muestra: Bilirrubina 5mg/dL GOT: 2600 UI GPT 2721Ul Anticuerpos IgG
anti-VHA positivos Anti-HBc IgM positivo Anti-VHC Anti-VHD positivos Antígeno
Australia positivo. ¿Cuál es el diagnóstico más probable?
a) Infección simultánea VHD más VHB en paciente portador de VHC.
b) Reactivación hepatitis C crónica.
c) Reactivación hepatitis B por presencia de VHD.
d) Hepatitis A aguda en paciente portador de VHC.
e) Se descarta infección por virus hepatotrópos.

Respuesta correcta: A

109. Paciente masculino de 64 años con antecedentes de ser fumador de 50 paquetes/año y


hábito enólico moderado ya que consume una botella de vino al día. Además el paciente
presenta antecedentes de EPOC moderadamente controlada en tratamiento con bromuro de
tiotropio y con salbutamoi inhalados. El paciente acude a su consulta por presenta disfagia
progresiva para sólidos de 3 meses de evolución. Además el paciente refiere que presenta
dolor retroesterneal leve intermitente con las comidas. Ha perdido en estos meses unos 10 kg
de peso (22 lb) y presenta cansancio. Nos informa de que presenta sialorrea sobre todo
nocturna y heces más oscuras de lo habitual. En la exploración física encontramos la presencia
de signo del pliegue positivo con deshidratación de mucosas oral y conjuntival. Además
presenta exceso cutáneo abdominal y a nivel de muslos y piernas por la pérdida de peso ¿Con
cuál de estas pruebas diagnosticará usted mejor a este paciente?
a) PHmetría ambulatoria de 24h.
b) Esofagoscopia con biopsia.
c) Manometría esofágica.
d) Tránsito esofágico con contraste.
Respuesta correcta: B

110. Un chico previamente saludable de 25 años de edad es llevado al departamento de


emergencias después de caerse de un árbol. Estaba construyendo una casa en el árbol
cuando perdió el equilibrio y cayó unos 3 m (10 p es) y aterrizó en su lado izquierdo. El
paciente no perdió el conocimiento, pero comenzó a experimentar dolor abdominal y de pecho
izquierdo. Él también tiene dolor en el hombro. La presión arterial es 103/71 mm Hg y el pulso
es 116 latidos por minuto. El examen físico muestra hematomas en la pared izquierda del tórax.
Los sonidos cardíacos son normales sin murmullo. El paciente informa un dolor agudo en el
lado izquierdo del tórax con una inspiración profunda, pero tiene el mismo murmullo vesicular a
la auscultación en ambos lados. Hay dolor en el margen costal izquierdo y en el cuadrante
superior izquierdo del abdomen con protección. Los ruidos intestinales son normales. El rango
de movimiento del hombro izquierdo es normal. Su hemoglobina es de 11.8 mgldL y una
radiografía de tórax de visión única es normal. La evaluación enfocada con ecografía no
muestra un líquido intraperitoneal libre significativo. ¿Cuál de los siguientes es el siguiente
paso más apropiado en el manejo de este paciente?
a) Monitorear con exámenes físicos seriados.
b) Obtener una tomografía computarizada del abdomen con contraste.
c) Obtener radiografías de parrilla costal.
d) Obtener una resonancia magnética del hombro izquierdo.
e) Realizar un lavado peritoneal de diagnóstico.
f) Realizar una laparotomía exploradora urgente.

Respuesta correcta: B

111. Un paciente masculino de 65 años en tratamiento con antiinflamatorios no esteroideos por


presentar antecedente de presentar artrosis, acude a Urgencias por un cuadro de dolor
abdominal de inicio brusco. El paciente refiere que, aunque ha tenido cuadros de molestias y
de dolor abdominal este es de mayor intensidad. El paciente no presenta otros antecedentes
personales médicos ni quirúrgicos. El paciente no consume tabaco. Bebedor de dos copas de
vino al día. No consumo de drogas ilícitas. Exploramos al paciente que presenta dolor a nivel
del epigastrio que irradia a fosa ilíaca derecha. En la auscultación encontramos ausencia de
ruidos hidroaéreos y presenta dolor a la palpación profunda con un abdomen en "tabla". El
examen de la sangre muestra leucocitosis y las pruebas de imagen no son concluyentes. ¿Cuál
sería la actitud terapéutica más adecuada?
a) Colocación de sonda nasogástrica, sueros y antibióticos.
b) Alimentación parenteral, omeprazol i.v, antibióticos.
c) Cirugía abdominal accediendo por incisión de Mc Burney.
d) Cirugía abdominal accediendo por laparotomía media.

Respuesta correcta:D

112. Una niña de 3 años es llevada al departamento de emergencia con letargo y fiebre por
diarrea durante varios días, y sus padres dicen que "de repente empeoró". La niña ha
rechazado líquidos durante las últimas 12 horas y no ha orinado hoy. Ella no tiene alergias y no
toma medicamentos. Se encuentra correctamente vacunada para su edad. Su temperatura es
de 39.7 °C (103.5 F), la presión arterial es de 60/28 mm Hg, y el pulso es de 145Iatidos por
minuto. En el examen, ella está letárgica y tiene una turgencia cutánea pobre. Su tiempo de
llenado capilar es de 5 segundos en el centro. A pesar de los numerosos intentos, no es posible
canalizar una línea intravenosa periférica y la condición continúa deteriorándose. ¿Cuál de los
siguientes es el mejor paso siguiente en el manejo de este paciente?
a) Intento de colocación de la línea arterial.
b) Intento de colocación del catéter venoso central.
c) Intento de canulación intraósea.
d) Intento de colocación de sonda nasogástrica.
e) Intento de colocación de la línea intravenosa periférica.
f) Transporte a la unidad de cuidados intensivos para la colocación del catéter venoso
central.

Respuesta correcta: C

113. Lee usted un estudio realizado para evaluar una nueva prueba para el diagnóstico precoz
de neumonías. Utiliza como patrón de referencia el cultivo del esputo, y al analizar los datos
obtenidos durante el estudio encontramos los siguientes resultados: 200 pacientes con cultivo
positivo (de los cuales 75 dieron positivo con el nuevo test), y 275 pacientes con cultivo
negativo (de los cuales 25 dieron positivo con el nuevo test). ¿Podría calcular el valor predictivo
negativo (VPN) de su nueva prueba?
a) 0.666.
b) 0.5.
c) 0.375.
d) 0.333.
e) 0.2.
f) 0.1.
g) 0.075.

Respuesta correcta: A

114. Una paciente femenina de 64 años acude a su consulta presentando incontinencia fecal
de 6 meses de evolución. La paciente tiene como antecedentes personales un cáncer de colon
derecho que fue intervenido mediante hemicolectomía derecha hace 7 años. Posteriormente
fue tratada con quimioterapia con remisión completa en el momento actual. Entre sus
tratamientos se encuentra únicamente ranitidina por presentar pirosis ocasional. Menarquia a
los 13 años de edad con menopausia a los 51 años de edad. Desde entonces no ha tenido
sangrados vaginales. 4 partos de los que 3 fueron por vía vaginal y uno por cesárea, además
de un aborto en el primer trimestre del embarazo. La paciente nos refiere que una gran
alteración en su calidad de vida. Realizamos una completa exploración física en la que
encontramos un esfínter anal externo competente. Se realiza una ecografía endoanal que
muestra integridad a nivel del esfínter anal sin lesiones asociadas que justifiquen la
incontinencia. ¿Cuál de los siguientes es un tratamiento correcto para el manejo de esta
paciente?
a) Fármacos peristálticos.
b) Laxantes prodiarreicos.
c) Electroestimulación.
d) Tratamiento quirúrgico

Respuesta correcta: C

115. Una paciente de 46 años acude a la clínica debido a hinchazón de la extremidad inferior
izquierda durante los últimos 2 años. Ella ha tenido varios episodios de celulitis que
involucraron la pierna izquierda y durante el episodio más reciente hace 6 meses, se sometió a
tratamiento con antibióticos por vía intravenosa que se complicó por una trombosis de la vena
exilar relacionada con el catéter que requirió de 3 meses de tratamiento anticoagulante. Su
historial médico también es significativo para la diabetes 'mellitus tipo 2 controlada con dieta. El
IMC es 37 kg/ m2. El examen físico muestra un edema firme de la extremidad inferior izquierda.
No hay eritema ni calor. El examinador no puede levantar la piel del dorso de los dedos del pie
izquierdo, pero puede hacerlo con los dedos del pie derecho. ¿Cuál de las siguientes es la
causa más probable de los síntomas de esta paciente?
a) Incompetencia de la válvula venosa.
b) Albuminuria
c) Linfedema.
d) Presión venosa central aumentada.
e) Esclerosis sistémica.

Respuesta correcta: B

116. Paciente masculino de 16 años de edad que presenta como antecedente médico una
enfermedad de Crohn con afectación colonice de 2 años de evolución. El paciente no presenta
otros antecedentes médicos ni quirúrgicos. Pesa 56 kg (123,5 lb) y mide 171 cm de altura. No
consume alcohol ni tabaco ni otras drogas ilícitas. Se encuentra en tratamiento de
mantenimiento con azatioprina. No consumo de otros fármacos. Acude a su consulta porque
hace 5 días refiere que comenzó la aparición de nódulos en ambas piernas. Durante la
exploración física evidenciamos en el paciente nódulos subcutáneos rojo violáceo, caliente,
doloroso con sensibilidad a la palpación de los mismo, de localización pretibial. bilaterales.
Además, el paciente refiere que presenta en esta última semana incremento del número de
deposiciones hasta 4 veces al día y dolor abdominal difuso que aumenta a la palpación.
Mantiene ruidos hidroaéreo presentes y el abdomen no presenta defensa en tabla tras la
palpación. Teniendo en cuenta la situación del paciente ¿Cuál sería su actitud en este caso?
a) Recomendar reposo relativo y paños calientes en ambas piernas y añadir tratamiento
antidepresivo.
b) Biopsiar zonas de piel alejados de las áreas lesionadas y pautar analgésicos opioides
de entrada
c) Sospechar la existencia de ura lesión tumoral maligna intestinal como desencadenante
del proceso cutáneo.
d) Ajuste del tratamiento de la enfermedad intestinal.
e) Laparotomía media por sospecha de invaginación intestinal.
f) Comienzo de tratamiento cutáneo para control de la enfermedad.

Respuesta correcta: D

117. Un paciente masculino de 45 años de edad acude a su consulta por presentar signos de
pirosis ocasional junto con tos frecuente. El paciente es fumador de una cajetilla de tabaco al
día. Consume una cerveza al día de forma ocasional. No más consumo de alcohol n, de otras
drogas. No presenta antecedentes médicos ni quirúrgicos de interés. Es bebedor de al menos 4
tazas de café al día. Refiere que siempre ha tenido tos pero que en el último mes es frecuente
la presencia de accesos de tos relacionado con un aumento de la sensación de pirosis. Torna
ocasionalmente omeprazol para el control de estos síntomas. En la auscultación encontramos
roncus difusos de predominio a nivel de los campos medios y pulmonares en ambos pulmones.
Los ruidos cardíacos son rítmicos y no aparecen soplos El paciente es diagnosticado de reflujo
gastroesofágico y usted decide la realización de una E sofagoscopia con toma de biopsias en
la que es diagnosticado de esófago de Barrett largo er un segmento de 6 cm sin encontrarse
displasia tras la evaluación de las biopsias. ¿Cuál es su tratamienot básico de elección?
a) Alta y nueva valoración si reaparecen nuevos síntomas
b) Tratamiento antibiótico erradicador del H. Pylorii.
c) Inhibidores de la bomba de protones de forma continuada.
d) Cirugía de extirpación de la zona lesionada.
e) Evaluación de la infección por Hellicobacter pylorii.
Respuesta correcta: C

118. Paul es un paciente masculino de 27 años con antecedentes de colitis ulcerosa. Se


encuentra en tratamiento con corticoesteroides orales a altas dosis, pero la enfermedad es
refractaria a este tratamiento por lo que acude a su consulta. El paciente no tiene otros
antecedentes médicos ni quirúrgicos y tiene buen seguimiento del tratamiento de, la
enfermedad. Evalúa al paciente que presenta gran afectación del estado general. Nos refiere
que presenta dolores contínuos a nivel de la región anal y en región de hipogastrio. Además,
presenta 3-4 deposiciones diarias con molestias en las mismas y secreción de moco. En la
exploración física encontramos dolor a la palpación a nivel de hipogastrio. El signo de
Blumberg es negativo. El paciente presenta presión arterial de 105/75 mmHg, frecuencia
cardíaca de 66 latidos por minuto y su temperatura exilar es de 36,2 °C (97,2 F). Ante el mal
estado de control que presenta el paciente se decide modificación a nivel.del tratamiento del
paciente. Señale cuál de los siguientes fármacos propondría como alternativa:
a) Ciclosporina,
b) Azatioprina.
c) Metotrexate.
d) Talidomida.

Respuesta correcta: A

119. Una paciente femenina de 81 años acude a su consulta por astenia franca durante el
último mes. Entre sus antecedentes destaca que la paciente es hipertensa y además presenta
hipercolesterolemia por lo cual recibe un inhibidor del enzima convertidor de la angiotensina y
atorvastatina de forma diaria. Además, la paciente padece dolores osteomusculares
generalizados atribuidos a artrosis que ella trata de forma espontánea con ácido acetilsalicílico.
La paciente no toma otros fármacos. No refiere tos ni molestias digestivas. En la exploración
física encontramos ruidos cardíacos rítmicos con frecuencia cardiaca baja y sin soplos. El
murmullo vesicular se encuentra conservado sin ruidos sobreañadidos. La exploración
abdominal no demuestra masas. No existe dolor a la palpación ni respuesta abdominal con
presencia de tabla a la palpación. Presión arterial de 95/60 mmHg con frecuencia cardíaca de
56 latidos por minuto. Encontramos palidez a nivel de la piel y a nivel de sus conjuntivas. La
paciente refiere que su hábito deposicional es normal y nunca ha visto sangre en las
deposiciones. La analítica revela: Hemoglobina 10g/dL Hematocrito 29% VCM 71 fl
Sideremia 15 mircrog/dL Una prueba de sangre oculta en heces da resultado positivo. Usted
índica una endoscopia digestiva alta y una colonoscopia total que no muestra lesión alguna.
¿Cuál sería la conducta más adecuada?
a) Indicar una exploración del intestino delgado con cápsula endoscópica.
b) Indicar una arteriografía selectiva de tronco celíaco y ambas mesentéricas para
descartar angiodisplasia.
c) Indicar una gammagrafia con Pertecnetato de Tc99 para descartar divertículo de
Meckel.
d) Prohibir el uso de ácido acetilsalicílico u otros AINE, dar hierro oral y repetir la
analítica en un plazo de 2 meses.
e) Utilización de una cápsula endoscópica.
f) Realizar una tomografía computarizada abdominal con contraste.

Respuesta correcta:D
120. Un hombre de 55 años acude al médico con una historia de 6 meses de una presión
torácica subesternal periódica. El ha experimentado esta opresión mientras camina colina
arriba o sube 2 tramos de escaleras. Sus antecedentes médicos son importantes por
hiperlipidemia y su medicación incluye estatina. Fuma 1 paquete diario y consume alcohol
ocasionalmente. Su presión arterial es 142/88 mmHg y pulso es 75/min. Electrocardiograma en
reposa se reporta normal. La prueba de esfuerzo muestra una depresión horizontal del
segmento ST en las derivadas II, III y aVF al 73% de la frecuencia cardiaca máxima predicha.
El ecocardiograma de esfuerzo muestra función sistólica ventricular izquierda normal al reposo
con hipocinesia de la pared inferior al pijo de ejercicio. En adición a la dosis de aspirina baja y
nitroglicerina sublingual a necesidad, cuál de los siguientes es la mejor terapia inicial para tratar
la condición actual de este paciente.
a) Amlodipina
b) Clortalidona
c) Dinitrato de isosorbide
d) Lisinopril
e) Metoprolol
f) Ranolazina

Respuesta correcta:E

121. Un hombre de 55 años de edad es traído a emergencias por repentina instauración de


palpitaciones y sensación opresiva torácica. Sus antecedentes médicos incluyen hipertensión,
gota y diabetes tipo 2. El monitor cardiaco muestra fibrilación atrial con una frecuencia de
120-140/min. Su presión inicial es de 112/70mmHg y la saturación de oxígeno 92% aire
ambiente. Mientras la enfermera intenta lograr un acceso venoso, el paciente cae en un estado
en el que no responde a las preguntas de la enfermera. No hay pulso palpable sobre las
carótidas o las arterias femorales e inicia una respiración agónica. El monitor cardiaco aún
muestra la fibrilación atrial en el mismo rango de frecuencia. ¿Cuál de los siguientes es el
mejor próximo paso a seguir en el manejo del paciente?
a) Análisis de gases arteriales
b) Compresiones torácicas
c) Desfibrilación
d) Lidocaina IV
e) Cardioversión sincronizada

Respuesta correcta:B

122. Una mujer de 38 años acude a urgencias por dolor torácico en las últimas horras. Ella lo
describe como punzante, localizacion central, no irradiado. Se empeora cuando inspira y con el
movimiento. Niega fiebre, escalofrío, disnea, edema o dificultad para respirar en la noche. Ella
refiere que el dolor apareció por primera vez cuando se ejercitaba hace 3 semanas, pero lo ha
sentido también en reposo. Refiere que se ha ejercitado con intensidad en búsqueda de perder
peso. Su IMC es 34 kg/m2. Su padre tuvo un infarto miocárdico a la edad de 60. Sus
antecedentes médicos de importancia: 2 partos vaginales normales. Refiere fumar medio
paquete de cigarrillos diarios. Al examen físico su presión arterial es 112/69 mmHg, frecuencia
cardiaca 72/min, El examen cardiológico revela S1 y S2 regulares, sin sonidos adicionales ni
soplos. Hay dolor a la palpación sobre el esternón. ¿Cuál es la causa más probable de su dolor
torácico?
a) Enfermedad por reflujo gastroesofágico
b) Embolismo pulmonar
c) Angina inestable
d) Pericarditis
e) Costocondritis
f) Angina estable
g) Desorden psiquiátrico

Respuesta correcta: E

123. Un hombre de 62 años visita a su médico familiar por dolor generalizado y malestar. Niega
asociación con fiebre, cefalea, dolor torácico o abdominal. Sus antecedentes destacan un
infarto de la pared inferior hace 6 años. Sus otros problemas médicos incluyen: hipertensión,
diabetes, hipercolesterolemia y gota. Sus medicamentos actuales son: aspirina, losartan,
naproxeno, atenolol, glipizide, colchicina y simvastatina. Al examen físico está afebril, presión
arterial 130/90 mmHg, pulso 80/min, respiración 18/min. Los laboratorios incluyen:
Sodio: 140 mEq/L Potasio: 4.2 mEq/L Bicarbonato: 21 mEq/L Cloro: 100 mEq/L Nitrógeno de
urea: 30 mg/dl Creatinina: 1.6 mg/dl AST: 113 IU/L ALT: 120 UI/L CPK: 14,998 mg/dl ¿Cuál es el
mejor próximo paso en el manejo de este paciente?
a) Suspender losartan
b) Suspender simvastatina
c) Iniciar N-Acetilcisteina
d) Ordenar pruebas hepáticas completas
e) Obtener biopsia hepática

Respuesta correcta: B

124. Un hombre de 62 años acude a urgencias debido a dolor subesternal y diaforesis. Tiene
una alta probabilidad pre-test de infarto miocárdico debido a hipertensión, diabetes, uso de
tabaco y antecedentes familiares. Un EKG muestra un patrón de bloqueo de rama izquierdo,
enzimas cardiacas ligeramente elevados. Se inicia terapia trombolítica intravenosa y se repito
el EKG que continúa mostrando el patrón de bloqueo de rama, pero el paciente está sin dolor
torácico y hemodinámicamente estable luego de la trombolisis. 2 días después desarrollo
malestar torácico y dificultad respiratoria. En la evaluación por un potencial re-infarto
miocárdico, ¿cuál de las siguientes es la prueba diagnóstica más apropiada?
a) Creatinin kinasa
b) Cambios EKG dinámicos
c) Deshidrogenasa láctica
d) Niveles de mioglobina
e) Niveles de troponina

Respuesta correcta: A

125. Un internista está discutiendo los riesgo de enfermedad cardiovascular con uno de sus
pacientes. El paciente tiene 40 años y fuma 20 paquetes año de cigarrillos. Mide 5pies 10
pulgadas y pesa 190 lbs. Tiene hipertensión leve que está siendo tratada efectivamente con
hidroclorotiazida. Su perfil lipídico es: Colesterol total 200mg/dl LDL: 160mg/dl HDL 35 mg/dl
¿Cuál de las siguientes es el factor modificable más importante en el paciente para reducir la
morbilidad y mortalidad relacionada a enfermedad cardiovascular?
a) Cesación de tabaco
b) Hipercolesterolemia
c) Mejorar la atención cardiaca si se diagnostica enfermedad arterial coronaria
d) Tratamiento más agresivo de la presión arterial
e) Reducción del peso

Respuesta correcta: A

126. Una mujer de 72 años con historia de hipertensión, diabetes mellitus y cáncer de mama
acude con empeoramiento de disnea. Ella fue diagnosticada con adenocarcinoma intraductal
de mama hace 3 años y actualmente está con terapia hormonal Al examen físico hay presión
arterial de 90/60 mmHg, pulso en 126/min, respiración en 20/min. La presión sistólica cae a
60mmHg durante la inspiración profunda. Hay distensión venosa yugular. Los sonidos
cardiacos se auscultan distantes. La radiografía de tórax muestra cardiomegalia, pero no hay
infiltrados o congestión venosa pulmonar. Electrocardiograma muestra bajo voltaje en varias de
las derivadas con elevación del ST en las derivadas Il y III. El ecocardiograma muestra efusión
pericárdica con colapso del atrio derecho y ventrículo derecho en diástole. ¿Cuál es el próximo
paso más apropiado a seguir en el manejo de la paciente?
a) Infusión ionotrópicos
b) Balón de contrapulsación aórtico
c) Diuréticos intravenosos
d) Reanimación con líquidos intravenosos
e) Pericardiocentesis

Respuesta correcta: E

127. Un hombre de 70 años, blanco es llevado a urgencias por paramédicos por queja de dolor
torácico que describe como aplastante 8/10 sin irradiación, nunca había sentido este dolor
antes. El paciente refiere que el dolor inició media hora después de una discusión con su
esposa. Refiere que padece de hipertensión controlada con metoprolol 50mg/d y lisinopril 20
mg/d, adicional toma 81 mg de aspirina por día. Durante la evaluación el paciente tuvo un
evento cerebrovascular isquémico hace dos meses sin déficit neurológico, también tuvo una
colecistectomia laparoscópica hace 8 días sin complicaciones. A la evaluación el paciente
parece ansioso, diaforético en estrés moderado. Sus signos vitales a la admisión son
temperatura de 37°C, presión arterial de 180/105 mmHg, pulso 110/min y respiración en 18/
min. La radiografía de tórax muestra leve cardiomegalia. EI EKG muestra taquicardia sinusal
con elevación del ST de 2mm en derivadas V1-V3 con hipertrofia ventricular izquierda. Las
enzimas cardiacas se solicitaron y están pendientes de resultado. El paciente se le administra
líquidos y 325 mg de aspirina. ¿Cuál de los siguientes es una contraindicación absoluta de
iniciar terapia trombolítica en este paciente?
a) Presión arterial de 180/105 mmHg
b) Cirugía reciente
c) Edad
d) Evento cerebral isquémico
e) Hipertrofia del ventrículo izquierdo

Respuesta correcta:D

128. Un hombre de 62 años tiene historia de 2 semanas de pesadez sub esternal y falta de aire
cuando camina subiendo las escaleras de su apartamento. El describe mareo y dificultar para
respirar durante el ejercicio en los últimos 6 meses que se alivia con el descanso. Sus
medicamentos incluyen simvastatina para la hipercolesterolemia y un inhalador de albuterol
para asma leve intermitente. Al examen físico se le encuentra un soplo mesosistólico 2/6
localizado en el segundo espacio intercostal derecho con irradiación a cuello. Hay retardo con
disminución del ascenso carotideo; división o separación paradójica del sonido S2 cardiaco. No
hay edema periférico, los pulsos son iguales en todas las extremidades. Su presión arterial es
130/95 mmHg bilateral, pulso en 80/min, temperatura de 37.2°C, respiración en 18/min. La
radiografía de tórax muestra un aumento en la relación cardiotorácica, una sombra ventricular
izquierda acentuada, y un leve borramiento del ángulo costo diafragmático en la vista lateral. El
electrocardiograma muestra una frecuencia y ritmo normal con Hipertrofia de ventrículo
izquierdo. El ecocardiograma muestra calcificación de la válvula aortica bicúspide, gradiente
transvalvular de 55mmHg y un área valvular aórtica de 1cm2. ¿Cuál de los siguientes es
próximo paso en el manejo más efectivo para reducir la mortalidad?
a) Remplazo de válvula aórtica
b) Profilaxis endocarditis
c) Valvuloplastia percutánea con balón
d) Furosemida
e) Metoprolol

Respuesta correcta: A

129. Un hombre de 68 años acude a urgencias vía ambulancia vía 911. Se queja de un dolor
sordo en el centro del pecho y sensación de opresión mientras iba manejando su carro de la
casa al trabajo. También refiere falta de aire. Refiere que había tenido episodios similares en el
pasado pero nunca tan severo. Ha notado que el dolor torácico ha ido empeorando
progresivamente en las últimas 2 semanas, especialmente cuando caminaba hacia su carro, y
que ahora estaciona más cerca de la oficina para evitar el tener que caminar. Es un fumador de
20 paquetes / año y bebe aproximadamente 20 unidades de alcohol por semana. Sus signos
vitales son: 37.4°C, pulso 115/min, presión arterial 150/96 mmHg. Al examen físico, tiene
sudoración profusa y gran disconfort. Actualmente respira oxígeno 100% a través de máscara
facial. No hay evidencia de ingurgitación yugular. A la evaluación revela sonidos cardiacos
distantes que son regulares en frecuencia y ritmo. El EKG no muestra elevación del segmento
ST. El paciente es llevado a cateterismo cardiaco y se realiza angiografía coronaria. La imagen
muestra oclusión del 70% de la arteria coronaria principal izquierda. ¿Cuál es el próximo mejor
paso a seguir en el manejo?
a) Cirugía de revascularización coronaria (bypass coronario)
b) Intervención coronaria percutánea
c) Recetar Atorvastatina
d) Ecocardiograma de esfuerzo

Respuesta correcta:A

130. Una mujer de 78 años acude a urgencias por dificultad respiratoria. En las últimas 3 horas
ha experimentado disnea incluso al reposo y tos con producción de esputo. Niega dolor
torácico. No hay antecedente de tuberculosis. Al examen físico hay una paciente con distress
respiratorio y frecuencia respiratoria en 32 / min, pulso en 140/min, presión arterial de 150/85
mmHg. Mientras espera ser evaluada nuevamente la paciente tos varias veces esputo
espumoso de color rosado. No hay dolor en la parrilla costal, sin embargo, la auscultación
revela estertores finos en todo el pecho. No hay soplos ni frotes. ¿Cuál es el próximo mejor
paso en el manejo del paciente?
a) Administración de diuréticos de asa, nitratos y morfina
b) Administrar hidralazina
c) Administrar verapamilo
d) Ecocardiograma
e) Electrocardiograma
Respuesta correcta:E

131. Un hombre de 46 años acude a urgencias con dificultad respiratoria e incomodidad


torácica. Su dolor empeora con la inspiración pero no se irradia. El paciente refiere que nunca
ha tenido una molestia como esta. No tiene antecedentes patológicos previos. Trabajo como
conductor de camión articulado grande. Al examen físico su presión arterial es de
110/70mmHg, su frecuencia cardiaca es de 110/min, su respiración de 31/min y su temperatura
es de 36.7°C. Electrocardiograma muestra taquicardia sinusal sin cambios en la onda T ni
datos isquémicos en el segmento ST. ¿Cuál de los siguientes es el diagnóstico más probable
de este paciente:
a) Disección de la Aorta ascendente
b) Infarto del Miocardio
c) Pneumotorax
d) Embolismo Pulmonar
e) Efusión Pleural

Respuesta correcta:D

132. Un paciente masculino de 50 años presenta poliuria y polidipsia. Ha fumado 1 paquete de


Cigarrillos al día durante los últimos 30 años. Él niega tener problemas médicos pasados Su
madre y un tío materno son diabeico3 Su altura es de 5"8 (172 cm), pesa 180 lb (81,6 kg), la
temperatura es de 37 ° C (98,6 F) el pulso es de 75 latidos por minuto, la presión arterial es de
150/90 mm Hg y la espiración es de 15 respiraciones por minuto. El panel de bioquímica
muestra Sodio 140 mE q/L Potasio 4.1 mEq/L Bicarbonato 26 mEq/L Glucosa en sangre
210mg/dL BUN 12 mg/dL Creatinina 0.9 mg/dL El paciente es diagnosticado de diabetes mellitu
tipo 2 Se le recomienda ejercicio modificación de la dieta. Se lo deriva a un oftalmólogo y se ID
examina adecuadamente para detectar la retinopatía diabética. ¿Cuál es la prueba más
sensible para detectar nefropatía en este paciente?
a) Prueba de la orna Orina para la relación de microalbúmina/creatinina.
b) Ultrasonido renal.
c) Prueba de tolerancia oral a la glucosa
d) TAC abdominal

Respuesta correcta: A

133. Un chico de 23 años acude al centro de atención orinaría debido aluna lesión en la rodilla.
Estaba jugando al baloncesto ayer cuando colisionó con un jugador contrario mientras corría a
gran velocidad. El paciente dejó de jugar debido al color, pero pudo caminar con moderada
incomodidad y pudo llegar a casa solo. Desde entonces, el dolor ha sido persistente y solo
parcialmente aliviado con ibuprofeno. En el examen, hay una leve hinchazón de los tejidos
blandos de la rodilla izquierda sin un derrame importante de la articulación. Hay sensibilidad en
la línea articular medial. El rango de movimiento de la rodilla es normal, sin crepitaciones. La
prueba de estrés por abducción (valgo) muestra cierta laxitud en la rodilla izquierda, pero está
limitada por el dolor. ¿Cuál de los siguientes es el diagnóstico más probable?
a) Fractura de la meseta tibial.
b) Lesión del ligamento cruzado anterior.
c) Lesión del ligamento colateral lateral.
d) Lesión del ligamento colateral medial.
e) Desgarro del menisco medial.
f) Tendinitis rotuliana.
Respuesta correcta:C

134. Un paciente masculino de 45 años tras aquejar pirosis y regurgitación casi diaria en los
últimos tres meses acude a su consulta. El paciente es fumador de 8 cigarrillos al día desde la
adolescencia. No consume alcohol porque le provoca un aumento de los síntomas; qua
presenta. Es bebedor habitual de café tanto en el desayuno corno por la tarde. No presenta
antecedentes médicos de interés. El paciente consume ranitidina de forma habitual para calmar
su sintomatología. Pesa 79 kg (174 lb) y mide 181 cm de altura, Tras su valoración clínica se
realiza una endoscopia a nivel esofágico y el paciente es diagnosticado de esofagitis por
reflujo. Se realiza un test para el diagnóstico del Helicobacter pylori en una biopsia antral y su
resultado es positivo. ¿Cuál sería, entre los siguientes, el tratamiento más adecuado?
a) Un inhibidor de la bomba de protonea adrninístrado a dosis estándar una vez al
día.
b) Un antagonista H2 a dosis doble administrado en dos dosis al día.
c) Cualquier antisecretor asociado a tratamiento antibiótico para erradicar la infección por
Helicobacter pylorii.
d) La asociación de un antagonista H2 con un procinétice.
e) Un procinético asociado a tratamiento antibiótico erradicador.

Respuesta correcta: A

135. Un niño de dos años es llevado a la consulta. A sus padres les preocupa que no sea tan
comunicativo como su hermana gemela. La hermana del paciente habla "constantemente" y
está empezando a juntar 3 palabras. El niño tiende a ser más callado y dice aproximadamente
70 palabras con fraSes de 2 palabras, ambos niños se inscribirán en preescolar en el otoño. Al
paciente le gusta hacer sonidos de pitido cuando juega con camiones de juguete, pero él no
juega con su hermana, solo a su lado. El niño está al día con las vacunas y sus parámetros de
crecimiento son promedio para su edad. Señala un juguete al otro lado de la habitación, mira a
su madre y sonríe, y luego trae el juguete para mostrarle. Cuando el médico se acerca, el
paciente corre para esconderse detrás de su padre e inicialmente se niega a cooperar o hacer
contacto visual. Cuando su madre trata de ayudarlo a subir a la mesa de examen, él la empuja
y dice: "Yo sí". ¿Cuál de los siguientes es el diagnóstico más probable en este paciente?
a) Trastorno del espectro autista.
b) Demora cognitiva.
c) Pérdida auditivo conductiva.
d) Retraso del lenguaje.
e) Desarrollo normal.

Respuesta correcta:E

136. Una paciente femenina de 76 años acude a consulta para valoración porque ingresó hace
4 meses por debut de insuficiencia cardiaca y fibrilación auricular. La paciente presenta
antecedentes personales de hipertensión arterial, y de diabetes mellitus no
insulinodependiente. Sus tratamientos incluyen amlodipino y metformina con buen control
evaluado por su médico de atención primaria. Acude para revisión. La paciente refiere
encontrarse mejor, sin cansancio y con menos edema de piernas, pero todavía no está del todo
bien. En su ecocardiograma encontramos disfunción ventricular izquierda moderada y el
tratamiento al alta tras el ingreso encontramos que se suspendió el amlodipino y se inició
furosemida y enalapril. A la exploración física comprueba que no ,existen signos de
insuficiencia cardiaca, pero llaman la atención unas cifras de presión arterial y, frecuencia
cardiaca de 160/90 mmHg y 125 latidos por minuto, respectivamente con una frecuencia
respiratoria de 16 respiraciones por minuto. Temperatura axilar de 36,1 °C (97 F). El ECG
muestra una arritmia con ausencia de ondas P y complejos QRS estrechos con una frecuencia
en torno a los 120 latidos por minuto. ¿Qué tratamiento es fundamental en esta paciente para
mejorar los síntomas y reducir la mortalidad por eventos cardiovasculares?
a) Digoxina y ácido acetilsalicílico.
b) Atenolol y ácido acetilsalicílico.
c) Verapamilo y acenocumarol
d) Carvedilol y acenocumarol.

Respuesta correcta:D

137. Un paciente de 27 años de edad sufre un accidente de circulación con traumatismo


pélvico por lo que es trasladado de urgencias al departamento de emergencias de su hospital.
En la realización del ECO Fast durante su traslado se objetiva una importante hemorragia
retroperitoneal. El paciente no presenta antecedentes, médicos de interés y fue intervenido a la
edad de 14 años por un cuadro de apendicitis tratado de forma laparoscópica con buena
evolución posterior. No consume ningún fármaco. No consumo de alcohol ni de tabaco. No
consumo de otras drogas ilícitas. Es intervenido en situación de shock hemodinámico con una
presión arterial de 60/30 mmHg y una frecuencia cardíaca de 126 latidos por minuto. Durante la
cirugía se objetiva y se realiza la reparación quirúrgica de ruptura de aorta abdominal. Tras la
misma el paciente permanece con aspiración nasogástrica y evoluciona sin problemas durante
tres días con una presión arterial de 110/80. Al cuarto día la diuresis disminuye y una analítica
sanguínea muestra: Urea de 169 mg/dl Creatinina de 2.7 mg/dl. Osmolaridad en orina 650
mosm/I Sodio en orina 10 mEq/I Relación urea orina/urea plasma de 18 Relación creatinina en
orina/creatinina en plasma de 70 La conducta más adecuada para tratar su situación será:
a) Ecografía abdominal inmediata para descartar uropatía obstructiva por hematoma
retroperitoneal.
b) Aumentar el aporte líquido i,v a 5 I/día, incluyendo transfusión de hematies
concentrados si fuera preciso.
c) Arteriografía renal para descartar obstrucción arterial renal.
d) Pautar Manitol y furosemida para que la diuresis retorne a la normalidad.
e) Evaluación urgente para entrada en lista de transplante renal.

Respuesta correcta: B

138. Paciente masculino de 34 años que ingresa en su hospital tras ser encontrado comatoso
en la calle. Evaluamos su historia clínica en nuestro hospital :/ encontramos que el paciente
presenta antecedentes de epilepsia y adicción a drogas no parenterales. El paciente está en
seguimiento por el servicio de Neurología de nuestro hospital y en tratamiento con valproato.
NO presenta antecedentes quirúrgicos de interés. Al ingreso se encuentra estuporoso, sin
focalidad neurológica, siendo imposible la realización de una anamnesis más extensa. La
exploración cardiovascular es normal. Tensión arterial de 138/78 mm Hg con frecuencia
cardíaca de 86 latidos por minuto. Oximetría con gafas nasales de oxígeno del 97%. Presenta
dolor difuso a la compresión en miembros superior e inferior derechos, con pantorrilla derecha
caliente y edematosa. Tras sondaje vesical se recuperan 200 mL de orina oscura. raliza una
analítica que presenta: Hemoglobina 14 4 g/dl Leucocitos 7.800/mm3 Glucemia 68 mg/dl Urea
114 mg/dl Creatinina 4,4 mg/dl Ácido úrico 9,9 mg/dl Calcio 7,0 mg/dl En la analítica de orina
encontramos: Densidad 1012 pH 5,5, Proteinuria +, Sedimento normal, Sodio urinario 64 mmol/
1 Se decide realizar una prueba de imagen de manera que se realiza una ultrasonografía renal
que es normal. ¿A cuál de las siguientes pruebas o determinaciones analíticas le ve más
utilidad inmediata para identificar la causa de la insuficiencia renal de este paciente?
a) Niveles de anticomicialcs en sangre
b) Nivel de creatinkinasa (CPK) en sangre
c) Patrones de citolisis y colostasís hepática.
d) Hemocultivos seriados y urocultivo.
e) Ecografía uretral.

Respuesta correcta: B

139. Acude a su consulta un paciente masculino de 66 años, de origen panameño, que


consulta por malestar general. Entre sus antecedentes de interés encontramos bronquitis
crónica en tratamiento con inhaladores de bromuro de tiotropio y salbutamol de forma diaria, e
hipertensión arterial en tratamiento con diuréticos de asa. Se encuentra en control por parte de
su médico de atención primaria con buen seguimiento del tratamiento farmacológico. El
paciente es exfumador de más de una cajetilla de tabaco al día aunque abandonó el hábito
hace diez años. No consumo de alcohol. El paciente refiere que desde hace un mes presenta
astenia y debilidad, además de una afectación generalizada del estado general. Su presión
arterial es de 135/95 mmHg con frecuencia cardíaca de 86 latidos por minuto. Temperatura
axilar de 34,8 °C (96,4 F). En la exploración física usted objetiva adenopatías dispersas en
región exilar bilateral y a nivel de ambas regiones inguinales, con hepatomegalia de tres
traveses de dedo y esplenomegalia asociadas. En la analítica que se le realiza se aprecia
hipogammaglobulinemia y leucocitosis de 42.000/mcL con línfocitosis superior al 80%. También
presenta ligera anemia y trombopenia. ¿Cuál sería su sospecha inicial?
a) Leucemia mieloideaguda.
b) Leucemia mielo.de crónica
c) Leucemia linfoide aguda.
d) Leucemia linfática crónica.
e) Linfoma de Burkitt
f) Mielofibrosis.

Respuesta correcta: D

140. Paciente masculino de 8 años que acude trasladado al departamento de emergencias de


su hospital por púrpura y equimosis importante, junto con fiebre de 5 días de evolución. El niño
no presenta antecedentes médicos ni quirúrgicos de interés. Está correctamente vacunado
para su edad. Presenta un peso y una talla en percentil 60 y 70 respectivamente para su edad.
La madre refiere que la temperatura ha ido aumentando en estos 5 días hasta más de 38,5w✓
(101,3 F) y que ha aparecido las lesiones cutáneas. En la exploración física destaca palidez
mucocutánea, cuadro purpúrico generalizado en el paciente, además de adenopatías
inguinales bilaterales y esplenomegalia palpable. En la auscultación pulmonar presenta
estertores en base derecha. La auscultación cardíaca es normal con ruidos cardíacos rítmicos
con frecuencia cardíaca aumentada. Presión arterial de 95/60 mmHg con frecuencia cardíaca
de 110 latidos por minuto. Se realiza una radiografía de tórax en la que encontramos patrón de
condensación en base derecha y ensanchamiento mediastíníco. La analítica es la siguiente: Hb
8 g/dL Leucocitos 45.000/mm3 con abundantes formas blásticas de tamaño pequeño con
escaso citoplasma Plaquetas 5.000/mm3. La médula ósea es muy rica en celularidad y un 42%
-Je la celularidac total tiene las mismas características que las formas blásticas de sangre
periférica. El diagnóstico de sospecha será:
a) Leucemia linfoblástica aguda.
b) Leucemia mieloblástica aguda
c) Leucemia linfática crónica.
d) Leucemia mieloide crónica.
e) Linfoma de Burkitt.
f) Linfoma de Hogdkin.
g) Linfoma de células peludas.
h) Tricoleucemia.

Respuesta correcta:A

141. Un paciente masculino caucásico de 36 años es llevado al servicio de urgencias debido a


debilidad de sus extremidades superiores e inferiores. El, examen neurológico revela debilidad,
atrofia, fasciculaciones, espasticidad e hiperreflexia de los músculos involucrados. Sus
funciones sensoriales, intestinales, vesicales y cognitivas están intactas. La creatina quinasa
sérica es normal. El examen del líquido cefalorraquídeo está dentro de los límites normales. La
electromiografía muestra denervación parcial crónica. El paciente es posteriormente
diagnosticado de esclerosis lateral amiotrófica. ¿Cuál de los siguientes ha sido aprobado por la
FDA y la EMA para su uso en pacientes con esclerosis lateral amiotrófica?
a) Riluzol.
b) Corticosteroides.
c) Inmunoglobulinas intravenosas.
d) Plasmaféresis.
e) Donepezil.

Respuesta correcta: A

142. Una niña de 6 meses, anteriormente sana, es llevada al médico por su madre por un
"cruce de ojos" ocasional. Su madre dice que los síntomas han empeorado, especialmente
antes de acostarse. La paciente nació por cesárea a las 37 semanas de gestación y ha
cumplido todos los hitos del desarrollo. Las vacunas de la paciente están al día. Se encuentra
en el percentil 50 de longitud y peso. Su temperatura es de 36,7°C (98°F), su pulso es de 130/
min, su respiración es de 40/min y su presión arterial es de 90/60 mm Hg. La agudeza visual es
de 20/20 en ambos ojos. Hay un reflejo luminoso asimétrico en la córnea. Cuando se cubre el
ojo izquierdo, el ojo derecho se mueve lateralmente. El resto de la exploración no muestra
anomalías. ¿Cuál de los siguientes es el siguiente paso más apropiado en la gestión?

A) Cirugía de urgencia
B) Parchar el ojo derecho
C) Medición de presión intraocular
D) Aplicación de ciclopentolato gotas en ojo izquierdo
E) Tranquilizar y dar cita de seguimiento

Respuesta correcta: D

143. Un chico joven de 29 años se somete a una esplenectomía después de un accidente


automovilístico. El día 3 de su hospitalización, recibe una transfusión de concentrados de
glóbulos rojos. Minutos después de que comienza la transfusión, comienza a respirar con
dificultad. Esto rápidamente progresa a dificultad respiratoria y presión arterial baja, y el
paciente pronto pierde la conciencia. ¿Cuál es la causa más probable de su reacción a la
transfusión?
a) Reacción hemolítica aguda debida a incompatibilidad ABO
b) Respuesta de anticuerpos anamnésicos contra un antígeno menor de glóbulos rojos.
c) Reacción anafiláctica.
d) Sangre contaminada con bacterias.
e) Reacción a las citocinas almacenadas en el producto sanguíneo transfundido.

Respuesta correcta:C

144. Paciente masculino de 38 años que acude a su consulta por presentar desde hace un mes
un cuadro clínico de sudoración y fiebre en brotes. La temperatura refiere que ha llegado a
superar los 38 °C (100,4 F) en algunas ocasiones. El paciente refiere que además en las
últimas dos semanas ha tenido una pérdida de peso de 4 kg (8.8 lb). El paciente no presenta
antecedentes personales de interés. En el pasado presentó un cuadro de mononucleosis
infecciosa que se resolvió sin secuelas posteriores. No consume alcohol de manera habitual ni
es fumador. No consumo de drogas ilícitas. En consulta el paciente presenta una presión
arterial de 115/75 mmHg con frecuencia cardíaca de 96 latidos por minuto. Presenta una
temperatura de 37,4 °C (99,3 F). En la exploración física se encuentran aclenopatías palpables
a nivel cervical que se confirman en las pruebas de imagen junto con adenopatias
mediastínicas y retroperitoneales. Tras un estudio histológico se encuentran células de Reed-
Sternberg. En la biopsia de médula ósea no se objetiva infiltración por la enfermedad. Teniendo
en cuenta la enfermedad que presenta el paciente señale en qué estadío se encontraría:
a) Estadio III-A
b) Estadío III-B
c) Estadio IV-A.
d) Estadio IV-B.
e) Estadío V-A.
f) Estadio V-B.

Respuesta correcta:B

145. Paciente femenina de 63 años acude a su consulta por febrícula, sensación distérmica y
malestar desde hace un mes. La paciente presenta antecedentes personales de diabetes tipo 2
e hipertensión arterial. Sus tratamientos., incluyen enalapril, metformina y glipirida. La paciente
refiere que presentó una dudosa odinofagia que no mejoró a pesar de amoxicilina hace 15 dias
pautado por su médico de atención primaria. Previamente la paciente no había presentado
ningún cuadro como el actual. La paciente no es fumadora y consume una ó dos cervezas a la
semana. No consumo de drogas ilícitas. Durante la exploración encontramos
hepatoesplenomegalia palpable de dos traveses de dedo. La auscultación pulmonar es normal
y en la auscultación cardíaca encontramos un soplo cardiaco que no estaba presente en la
última revisión, hace 6 meses. Se realiza ecocardiograma que objetiva una verruga típica de
endocarditis en la válvula mitral, con hemocultivos negativos. En la analítica que s.s.. realiza
destaca la presencia de factor reumatoide positivo e hipocomplemetemia. ¿Cuál de entre los
siguientes es el diagnóstico más probable?
a) Tularemia.
b) Fiebre Q.
c) Brucelosis.
d) Rickettsiosi
e) Borreliosis.
f) Endocarditis "decapitada" por consumo previo de antibióticos.

Respuesta correcta:F

146. Paciente masculino de 58 años que vive en la calle y es trasladado por los servicios
sociales al departamento de emergencias de su hospital por cuadro de expectoración
marrónacea, fiebre y malestar general de 10 días de evolución. El paciente tiene antecedentes
de ingresos previos por presentar una hepatitis crónica de posible origen alcohólico. Es
bebedor habitual de dos litros de vino al día y fumador esporádico según la disponibilidad de
cigarrillos de hasta una cajetilla al día. No sigue control por su hepatopatía en el hospital ya que
no ha acudido a las citas establecidas. Presenta presión arterial de 140/105 mmHg con
frecuencia cardíaca de 85 latidos por minuto. Frecuencia respiratoria de 21 respiraciones por
minuto. La temperatura exilar es de 38,2 °C (1(10,8 F). En la auscultación pulmonar
encontramos roncus en el pulmón derecho con predominio en los campos inferiores. El esputo
es maloliente. A pesar de sus gafas nasales su oximetría presenta una saturación de oxígeno
del 92%. Se realiza una radiografía de tórax en que observa lesión nodular cavitada con nivel
hidroaéreo en lóbulo inferior derecho. De cara a iniciar tratamiento ¿Cuál es el germen más
habitualmente implicado en una situación como la que presenta el paciente?
a) Gram negativos.
b) S. pneumoniae
c) Legionella.
d) Anaerobios.
e) Acinetobacter

Respuesta correcta: D

147. Un paciente afroamericano de 35 años acude al médico quejándose de 2 semanas de


distensión abdominal. Él ha estado en un programa de rehabilitación de drogas durante 2 años
debido a abuso de heroína previo. Los perfiles recientes de hepatitis viral y VIH fueron
negativos. Su temperatura es de 37,1 °C (98.7 F), la presión arterial es de 145/82 mm Hg, el
pulso es de 80 latidos por minuto y las respiraciones son de 14 por minuto. El paciente es
obeso con un índice de masa corporal de 40 kg/m2. El examen muestra edema periorbitario,
ascitis y edema con fóvea ++ en ambas piernas hasta las rodillas. Los ruidos de la respiración
disminuyen en la base del pulmón derecho. La orina de 24 horas muestra una excreción
proteica de 7.5 g. ¿Cuál de los siguientes es más probable que esté presente en. la biopsia
renal?
a) Giomerulosclerosis focal segmentaria
b) Nefropatía IgA.
c) Nefropatía membranosa.
d) Enfermedad de cambios mínimos.
e) Amiloidosis.
f) Nefropatía por cadenas monoclonales.

Respuesta correcta: A

148. Una joven nuligravida de 29 años acude al médico porque no ha menstruado durante 6
meses. La menarquia fue a los 14 años. Tenía períodos irregulares entre los 14 y los 16 años, y
luego tomó anticonceptivos hormonales orales hasta el año pasado cuando se casó. La
paciente no conoce su historia familiar ya que es adoptada. Ella es maestra de física de la
escuela secundaria y entrena equipos de deporte. Ella corre recreativamente y come una dieta
equilibrada. Ella no bebe, fuma ni usa, drogas ilícitas. Sus signos vitales son normales y el IMC
es de 22 kg/m2. El examen físico no muestra anormalidades. La prueba de embarazo en la
orina es negativa. ¿Cuál de las siguientes e la prueba de detección más adecuada para esta
paciente?
a) Cariotipo.
b) Resonancia magnética de la hipófisis.
c) No hacer más estudios.
d) Suero 17-hidroxiprogesterona.
e) Prolactina sérica.
f) Ultrasonido de la pelvis.

Respuesta correcta: E

149. Paciente masculino de 15 años que acude a su hospital por náuseas, vómitos, molestias
abdominales y diarrea acuosa (5 episodios) que se inició hace 6 horas. El paciente es sano sin
antecedentes médicos de interés. En sus antecedentes quirúrgicos destaca haber sido
intervenido de una fractura supracondílea de húmero a los 8 años de edad. Está correctamente
vacunado. No consumo de tóxicos, alcohol ni tabaco. Dentro de su familia más cercana 4
familiares padecen síntomas similares después de haber comido juntos huevos y hamburguesa
de pollo. No presenta fiebre con presión arterial de 95/60 mmHg, frecuencia cardiaca de 65
latidos por minuto y temperatura en consulta de 35,9 °C (96,6 F). ¿Cuál es el agente etiológico
más probable de la infección?
a) Staphylococcus aureus
b) Salmonella.
c) Escherichia coli enterotoxigénica.
d) Rotavirus.
e) Shigella.
f) Vibrio.

Respuesta correcta:A

150. Un hombre de 48 años es traído por su familia al departamento de emergencias de su


hospital con disnea, agitación y tos productiva. Había tenido fiebre y escalofríos los dos últimos
días a los que no dio mayor importancia, pero bruscamente comienza con disnea hace una
hora. Los signos vitales son: presión arterial de 110/45 mmHg, frecuencia cardiaca de 126
latidos por minuto,37 respiraciones por minuto, saturación de oxígeno 88% y temperatura
39,6°C (103,3 F). Exploramos al paciente que presenta sudoración profusa y agitación. Las
extremidades se encuentran frías a la palpación. Presenta tos con expectoración de esputo
rosado. El pulso carotídeo es lleno y colapsante (pulso de Corringan) y en la auscultación
cardíaca presenta un soplo diastólico precoz. La auscultación pulmonar demuestra la presencia
de crepitantes húmedos generalizados en los dos pulmones y un descenso evidente del
murmullo vesicular. Además de la intubación oro traqueal urgente y administración de
furosemida intravenosa, ¿cuál de las siguientes acciones inmediatas es la más importante?
a) Administrar nitritos intravenosos.
b) Realizar radiografía de tórax y comenzar antibioterapia empírica con Ceftriaxona y
Levofloxacino.
c) Realizar ecocardiograma urgente.
d) Sacar hemocultivos y comenzar antibioterapia empírica con Cloxacilina, Gentamicina y
Ampicilina y realizar ecocardiograma urgente. En función de los resultados obtenidos
valorar la indicación de cirugía urgente.
e) Realizar un angioTC para evaluar necesidad de fibrinolisis.

Respuesta correcta:C

151. Un paciente masculino de 32 años acude a su hospital por presentar fiebre de aparición
en las últimas horas junto con confusión mental. El paciente acude junto con sus padres que no
saben qué ha podido pasar porque nunca antes había presentado un cuadro similar. El
paciente presenta antecedentes de cuadros de virus herpes simplex durante su infancia y
adolescencia uno de ellos causante de una úlcera corneal. No toma fármacos de forma
habitual. Los familiares niegan consumo de alcohol ni tabaco. No consumo conocido de drogas
ilícitas. Entre sus antecedentes no destaca ningún otro dentro del hospital. En el departamento
de urgencias de su hospital presenta una presión arterial de 95/60 mmHg, con frecuencia
cardíaca de 90 latidos por minuto. Temperatura axilar de 39,2 °C (102,5 F). Exploramos al
paciente que presenta lesiones cutáneas en pie izquierdo a nivel distal, maculares, de
milímetros de diámetro, de aspecto isquémico hemorrágico y la auscultación cardiopulmonar es
normal. A los pocos días se obtiene crecimiento de Staphylococcus aureus meticilin-sensible en
tres hemocultivos de tres obtenidos. ¿Cuál de las siguientes sería la actitud correcta a seguir
en ese momento?
a) Considerar el resultado de los hemocultivos como probable contaminación.
b) Pautar de inmediato tratamiento antibiótico con penicilina y gentamicina durante 10
días.
c) Comenzar tratamiento con cloxacilina y gentamicina y realizar estudio
ecocardiográfico por la existencia probable de endocarditis aguda.
d) Descartar la existencia de endocarditis por la ausencia de soplos en la auscultación
cardiaca y buscar focos de posible osteomielitis.

Respuesta correcta: C

152. Paciente femenina originaria de Guatemala de 40 años que acude para valoración a su
consulta. La paciente presenta antecedentes de digestiones pesadas, por lo que toma
habitualmente metoclopramida y omeprazol. No tiene otros antecedentes personales de
interés. No consume alcohol, tabaco ni otros fármacos. Presenta un IMC (Body Mass Index) de
25,3 kg/m2. En los últimos 5 años había tomado de forma ininterrumpida anticonceptivos orales
hasta hace 7 meses que los suspendió 5' desde entonces Está amenorreica. Previamente sus
menstruaciones habían sido siempre regulares, había quedado embarazada con facilidad hacía
14 y 10 años, y había dado lactancia materna durante 4 meses con sus dos hijos. ¿Qué actitud
inicial considera más correcta para investigar la amenorrea de esta mujer?
a) Solicita, una determinación en sangre de TSH tras TRH.
b) Solicitar una resonancia magnética nuclear de la hipófisis.
c) Solicitar una prueba de estimulación de LH, FSH tras GnRH
d) Suspender todo tipo de medicación y solicitar una determinación de prolactina.
e) Realizar una radiografía de cráneo

Respuesta correcta:D

153. Un paciente masculino de 31 años es llevado al departamento de emergencias después


de haber estado involucrado en una colisión de un vehículo de motor dónde viajaba como
pasajero. Le dieron 3 L de solución salina normal en la ambulancia en su camino al hospital y
ha estado recibiendo 51/min de oxígeno por cánula nasal. Él está agitado y mueve las cuatro
extremidades espontáneamente. Su presión arterial es de 85/55 mmHg y su frecuencia
cardíaca es de 120 latidos por minuto. Su frecuencia respiratoria es de 30 por minuto. Sus
pupilas son simétricas y reactivas a la luz. Sus venas del cuello son planas y su tráquea se
desplaza hacia la derecha.
Sobre el hemitórax izquierdo, los ruidos respiratorios están ausentes y la percusión es mate
¿Cuál de los siguientes diagnósticos es más probable?
a) Neumotórax por tensión.
b) Contusión pulmonar.
c) Ateiectasia pulmonar.
d) Hemotórax.
e) Rotura diafragmática.
f) Rotura traqueobronquial.
Respuesta correcta:D

154. Paciente femenina de 27 años, diagnosticada hace 3 años de asma persistente leve, en
tratamiento con corticoides inhalados y beta-2 agonista de acción corta a demanda, que acude
a urgencias por disnea, tos y sensación de opresión torácica. La paciente no presenta otros
antecedentes personales ni familiares de interés. Previamente la paciente ha tenido buen
control de su enfermedad con leves crisis de reagudizacion con buen control con inhaladores.
Durante su estancia en urgencias la paciente presenta una presión arterial de 125/85 mmHg,
frecuencia cardíaca de 105 latidos por minuto y frecuencia respiratoria de 23 respiraciones por
minuto. Durante la auscultación encontramos sibilancias intensas. Además de la taquipnea y
taquicardia en la inspección se observa tiraje. Tras su diagnóstico la paciente se ingresa en su
hospital. Al segundo día de ingreso hospitalario presenta dolor centrotorácico súbito irradiado a
cuello que se mantiene posteriormente. En la exploración física durante la palpación del cuello
presenta crepitación, Señale cuál es su sospecha:
a) Neumotórax.
b) Pericarditis aguda.
c) Neumomediastino.
d) Rotura traqueal.
e) Infarto agudo miocardio.
f) Taponamiento cardíaco.
g) Infarto agudo de miocardio.
h) Tromboembolismo pulmonar.
Respuesta correcta:C

155. Un paciente de 68 años con miastenia gravis se evalúa por debilidad progresiva mientras
está hospitalizado. El paciente ingresó 2 días antes debito a fiebre, tos productiva y dolor
pleurítico en el pecho. La radiografía de tórax mostró una consolidación del lóbulo inferior
derecho para la cual el paciente recibió ceftriaxona y azitromicina per vía intravenosa. También
ha estado recibiendo su dosis ambulatoria de piridostigmina. En las últimas horas, ha
experimentado una debilidad progresiva generalizada y una incapacidad para expectorar e
esputo. Su temperatura es de 37.6 °C (99.8 F), la presión arterial es de 130/70 mm Hg, el pulso
es de 110 latidos por minuto y la frecuencia respiratoria es de 25 respiraciones por minuto. La
saturación de oxígeno es del 89% con oxígeno a 4L/min por cánula nasal. El paciente parece
estar en apuros, y su patrón de respiración es rápido y superficial con ocasionales sonidos de
gorgoteo. La auscultación pulmonar reveía crujidos gruesos en todas partes. Hay una ligera
debilidad en las extremidades, pero los reflejos tendinosos profundos son normales. Su
capacidad vital es ahora de 1.0 L en comparación con 1.5 L en el momento de la admisión, y el
gas en la sangre arterial muestra: pH de 7.27 pCO2 de 55 mm Hg p02 de 60 mm Hg El
paciente es intubado y trasladado a la unidad de cuidados intensivos. ¿Cuál de los siguientes
es el mejor paso siguiente en el manejo de este paciente?
a) Observación cercana solo.
b) Aumento de la dosis de piridostigmina.
c) Atropina intravenosa.
d) Plasmaféresis.
e) Timectomía.
Respuesta correcta: D

156. Paciente masculino de 71 años que es trasladado por los servicios de emergencias a su
hospital po presentar dolor torácico. Entre sus antecedentes destaca que el paciente es
exfumador de 15 cigarrillos al día durante más de 40 años aunque abandonó el hábito hace 5
años, hipertenso y diabético. En sus tratamientos se encuentra la hidroclorotiazida, metformina,
glipirida y omeprazol ocasional para el tratamiento del reflujo gastroesofágico. Presenta un
control aceptable de las cifras de presión arterial y glucemia que es evaluado por su médico de
atención primaria. Consume una cerveza al día. No consumo de otras drogas ilícitas. Hace 4
meses presentó un accidente cerebrovascular con paresia residual en el brazo izquierdo. No
presenta anticoagulación ni antiagregación. Acude a su hospital con dolor precordial intenso de
2 horas de evolución. En la exploración física presenta piel fría y sudorosa, tensión arterial de
75/35 mmHg con frecuencia cardíaca de 125 latidos por minuto. Se realiza de forma urgente un
electrocardiograma en el que encontramos elevación marcada del segmento ST en las
derivaciones V1-V6, I y aVL. ¿Cuál de los siguientes tratamientos de reperfusión coronaria es
el más adecuado?
a) Fibrinolisis sistémica de forma inmediata.
b) Estabilización hemodinámica y fibrinolisis programada.
c) Revascularización percutánea.
d) Estabilización hemodinámica y procedimiento de revascularización diferido.

Respuesta correcta:C

157. Paciente masculino de 48 años que acude a su consulta por tos no productiva de 3 meses
de evolución. El paciente presenta antecedentes de hipertensión arterial así como un cuadro de
colecistitis por el que fue intervenido mediante colecsitectomia abierta tras ingreso hospitalario.
Se encuentra en tratamiento con lisinopril. Es fumador de una cajetilla de tabaco al dia desde la
adolescencia. No consume alcohol de forma habitual y solo esporádicamente' los fines de
semana. No consume otras drogas ilícitas. El paciente pesa 79 kg (174 lb) y mide 186 cm.
Práctica deporte de forma habitual con un grupo de amigos. Ante la clínica del paciente explora
al mismo y encuentra que en la auscultación el murmullo vesicular está conservado en el
pulmón derecho mientras que en los campos pulmonares superiores izquierdos está abolido.
La auscultación cardíaca es normal con ruidos cardíacos rítmicos sin soplos añadidos. La
vpresión arterial es de 115/70 mmHg con frecuencia cardíaca de 62 latidos por minuto. Usted
ecide realizar una radiografía simple de tórax en la que encontramos atelectasia del lóbulo
superior izquierdo. Además, se realiza una analítica rutinaria que muestra un potasio sérico de
4,2 mEq/L y un sodio sérico de 125 mEq/L. ¿Cuál de entre los siguientes es el diagnóstico más
probable?
a) Carcinoma epidermoide con metástasis suprarrenales.
b) Adenocarcinoma metastásico.
c) Carcinoma de células gigantes.
d) Carcinoma microcítico con secreción inadecuada de ADH (SIADH).
e) Carcinoma suprarrenal con metástasis pulmonar.

Respuesta correcta: D
158. Un niño de 5 meses es llevado al departamento de emergencias con un historial de 2 días
de vómitos proyectiles, diarrea y un episodio de actividad convulsiva tonicoclónica generalizada
justo antes de la llegada. Él ha sido exclusivamente alimentado con fórmula desde su
nacimiento. Nació a las 39 semanas de gestación y tuvo un "nacimiento natural" en casa. El
embarazo y el parto no fueron complicados. El niño no se sometió a una prueba de detección
de recién nacidos debido a las preferencias maternas. Sus padres y su hermana de 4 años
están sanos. Uno de sus tíos maternos es discapacitado intelectualmente y vive en un centro
de atención a largo plazo. El peso, la altura y la circunferencia de la cabeza están en el
percentil 10 del niño. El examen físico muestra cabello y piel claros, ojos azules, erupción
eccematosa y deshidratación leve. Su orina tiene un olor a humedad rancia. ¿Cuál de las
siguientes es la prueba de diagnóstico más adecuada para la condición de este paciente?
a) Actividad de la aldolasa.
b) Actividad de la galactosa-1-fosfato uridil transferasa.
c) Análisis cuantitativo de aminoácidos.
d) Hormona estimulante de la tiroides.
e) Cariotipo.

Respuesta correcta:C

159. Un anciano de 73 años acude a la consulta debido a 3 meses de urgencia urinaria


progresiva, nicturia y flujo urinario débil. No tiene fiebre, dolor abdominal, hematuria, malestar
general o pérdida de peso. Su único medicamento es lisinopril para la hipertensión esencial. El
paciente no tiene antecedentes de diabetes mellitus o enfermedad isquémica del corazón. Él
no usa tabaco, alcohol o drogas ilícitas. El examen rectal revela una próstata lisa, firme y
agrandada sin induración o asimetría. El examen neurológico es normal. El análisis de orina no
muestra proteinuria ni hematuria. La creatinina sérica del paciente es de 2.1 mg/dl., que es más
alta que su nivel basal de creatinina de 1.2 mg/dl hace 4 meses. El antígeno prostático
específico es normal. ¿Cuál de los siguientes es el siguiente paso más apropiado en
evaluación de la lesión renal aguda de este paciente?
a) Cistoscopia.
b) Biopsia de riñón.
c) Biopsia de próstata.
d) Ecografía renal.
e) Citología urinaria.

Respuesta correcta:C

160. Un paciente masculino de 18 años de edad acude a Urgencias por cefalea brusca
mientras realizaba esfuerzo jugando un partido de fútbol. EI paciente no había presentado
previamente ningún cuadro de dolor similar a este a lo largo de su vida. Acude acompañado de
sus padres al departamento de emergencias que nos cuentan que durante la infancia presentó
una crisis de convulsiones durante un cuadro febril causado por una otitis media serosa. No
presenta otros antecedentes médicos ni quirúrgicos. No es fumador ni consume alcohol. Niega
el consumo de drogas ilícitas. Niega haber tenido un traumatismo previo. Desde la aparición d
la cefalea y durante su estancia en el departamento de emergencias el paciente comienza con
pérdida de conciencia además de presentar en la exploración rigidez de nuca. Presenta
además náuseas, pero sin vómitos. La temperatura del paciente tomada en urgencias es de
35,8 °C (96,4 F). Se le realiza de forma urgente una tomografía computarizada craneal
diagnosticándose al paciente. Ante el cuadro que usted sospecha ¿Cuál es la causa más
frecuente de que se produzca esta situación?
a) Aneurisma.
b) Tumoración intracraneal.
c) Malformación arteriovenosa.
d) Discrasia sanguínea.
e) Émbolos de origen cardíaco.
f) Colonización por Aspergillus.

Respuesta correcta: A

También podría gustarte